mrcs preparation emrcs questions lowerlimb

136
A 20 year old lady presents with pain on the medial aspect of her thigh. Investigations show a large ovarian cyst. Compression of which of the nerves listed below is the most likely underlying cause? A. Sciatic B. Genitofemoral C. Obturator D. Ilioinguinal E. Femoral cutaneous Theme from April 2012 Exam The cutaneous branch of the obturator nerve is frequently absent. However, the obturator nerve is a recognised contributor to innervation of the medial thigh and large pelvic tumours may compress this nerve with resultant pain radiating distally. Obturator nerve The obturator nerve arises from L2, L3 and L4 by branches from the ventral divisions of each of these nerve roots. L3 forms the main contribution and the second lumbar branch is occasionally absent. These branches unite in the substance of psoas major, descending vertically in its posterior part to emerge from its medial border at the lateral margin of the sacrum. It then crosses the sacroiliac joint to enter the lesser pelvis, it descends on obturator internus to enter the obturator groove. In the lesser pelvis the nerve lies lateral to the internal iliac vessels and ureter, and is joined by the obturator vessels lateral to the ovary or ductus deferens. Supplies Medial compartment of thigh Muscles supplied: external obturator, adductor longus, adductor brevis, adductor magnus (not the lower part-sciatic nerve), gracilis The cutaneous branch is often absent. When present, it passes between gracilis and adductor longus near the middle part of the thigh, and supplies the skin and fascia of the distal two thirds of the medial aspect. Obturator canal Connects the pelvis and thigh: contains the obturator artery, vein, nerve which divides into anterior and posterior branches. Cadaveric cross section demonstrating relationships of the obturator nerve

Upload: faisol-kabir

Post on 07-Aug-2015

216 views

Category:

Health & Medicine


23 download

TRANSCRIPT

Page 1: MRCS preparation emrcs questions Lowerlimb

A 20 year old lady presents with pain on the medial aspect of her thigh. Investigations show a large ovarian cyst. Compression of which of the nerves listed below is the most likely underlying cause?

A. Sciatic

B. Genitofemoral

C. Obturator

D. Ilioinguinal

E. Femoral cutaneous

Theme from April 2012 Exam The cutaneous branch of the obturator nerve is frequently absent. However, the obturator nerve is a recognised contributor to innervation of the medial thigh and large pelvic tumours may compress this nerve with resultant pain radiating distally.

Obturator nerve The obturator nerve arises from L2, L3 and L4 by branches from the ventral divisions of each of these nerve roots. L3 forms the main contribution and the second lumbar branch is occasionally absent. These branches unite in the substance of psoas major, descending vertically in its posterior part to emerge from its medial border at the lateral margin of the sacrum. It then crosses the sacroiliac joint to enter the lesser pelvis, it descends on obturator internus to enter the obturator groove. In the lesser pelvis the nerve lies lateral to the internal iliac vessels and ureter, and is joined by the obturator vessels lateral to the ovary or ductus deferens. Supplies

• Medial compartment of thigh • Muscles supplied: external obturator, adductor longus, adductor brevis,

adductor magnus (not the lower part-sciatic nerve), gracilis • The cutaneous branch is often absent. When present, it passes between gracilis

and adductor longus near the middle part of the thigh, and supplies the skin and fascia of the distal two thirds of the medial aspect.

Obturator canal

• Connects the pelvis and thigh: contains the obturator artery, vein, nerve which divides into anterior and posterior branches.

Cadaveric cross section demonstrating relationships of the obturator nerve

Page 2: MRCS preparation emrcs questions Lowerlimb

A 22 year old man suffers a compound fracture of the tibia. During attempted surgical repair the deep peroneal nerve is divided. Which of the following muscles will not be affected as a result?

A. Tibialis anterior

B. Peroneus longus

C. Extensor hallucis longus

D. Extensor digitorum longus

E. Peroneus tertius

Peroneus longus is innervated by the superficial peroneal nerve (L4, L5, S1).

Page 3: MRCS preparation emrcs questions Lowerlimb

Image sourced from Wikipedia

Deep peroneal nerve Origin From the common peroneal nerve, at the lateral aspect of the

fibula, deep to peroneus longus Nerve root values L4, L5, S1, S2 Course and relation

• Pierces the anterior intermuscular septum to enter the anterior compartment of the lower leg

• Passes anteriorly down to the ankle joint, midway between the two malleoli

Page 4: MRCS preparation emrcs questions Lowerlimb

Terminates In the dorsum of the foot Muscles innervated

• Tibialis anterior • Extensor hallucis longus • Extensor digitorum longus • Peroneus tertius • Extensor digitorum brevis

Cutaneous innervation

Web space of the first and second toes

Actions • Dorsiflexion of ankle joint • Extension of all toes (extensor hallucis longus and extensor

digitorum longus) • Eversion of the foot

After its bifurcation past the ankle joint, the lateral branch of the deep peroneal nerve innervates the extensor digitorum brevis and the extensor hallucis brevis The medial branch supplies the web space between the first and second digits. hich of the following forms the medial wall of the femoral canal?

A. Pectineal ligament

B. Adductor longus

C. Sartorius

D. Lacunar ligament

E. Inguinal ligament

Theme from September 2011 Exam Theme from September 2012 Exam

Femoral canal The femoral canal lies at the medial aspect of the femoral sheath. The femoral sheath is a fascial tunnel containing both the femoral artery laterally and femoral vein medially. The canal lies medial to the vein. Borders of the femoral canal Laterally Femoral vein Medially Lacunar ligament Anteriorly Inguinal ligament Posteriorly Pectineal ligament Image showing dissection of femoral canal

Page 5: MRCS preparation emrcs questions Lowerlimb

Image sourced from Wikipedia

Contents

• Lymphatic vessels • Cloquet's lymph node

Physiological significance Allows the femoral vein to expand to allow for increased venous return to the lower limbs. Pathological significance As a potential space, it is the site of femoral hernias. The relatively tight neck places these at high risk of strangulation. You decide to take an arterial blood gas from the femoral artery. Where should the needle be inserted to gain the sample?

A. Mid point of the inguinal ligament

B. Mid inguinal point

Page 6: MRCS preparation emrcs questions Lowerlimb

C. 2cm inferomedially to the pubic tubercle

D. 2cm superomedially to the pubic tubercle

E. 3cm inferolaterally to the deep inguinal ring

Theme from September 2012 Exam The mid inguinal point in the surface marking for the femoral artery.

Femoral triangle anatomy Boundaries Superiorly Inguinal ligament Laterally Sartorius Medially Adductor longus Floor Iliopsoas, adductor longus and pectineus Roof • Fascia lata and Superficial fascia

• Superficial inguinal lymph nodes (palpable below the inguinal ligament)

• Great saphenous vein

Image sourced from Wikipedia

Page 7: MRCS preparation emrcs questions Lowerlimb

Contents

• Femoral vein (medial to lateral) • Femoral artery-pulse palpated at the mid inguinal point • Femoral nerve • Deep and superficial inguinal lymph nodes • Lateral cutaneous nerve • Great saphenous vein • Femoral branch of the genitofemoral nerve

A 34 year old man is shot in the postero- inferior aspect of his thigh. Which of the following lies at the most lateral aspect of the popliteal fossa?

A. Popliteal artery

B. Popliteal vein

C. Common peroneal nerve

D. Tibial nerve

E. Small saphenous vein

Theme from April 2012 Exam The contents of the popliteal fossa are (from medial to lateral): Popliteal artery Popliteal vein Tibial nerve Common peroneal nerve The sural nerve is a branch of the tibial nerve and usually arises at the inferior aspect of the popliteal fossa. However, its anatomy is variable.

Popliteal fossa Boundaries of the popliteal fossa Laterally Biceps femoris above, lateral head of gastrocnemius and plantaris below Medially Semimembranosus and semitendinosus above, medial head of

gastrocnemius below Floor Popliteal surface of the femur, posterior ligament of knee joint and

popliteus muscle Roof Superficial and deep fascia Image showing the popliteal fossa

Page 8: MRCS preparation emrcs questions Lowerlimb

Image sourced from Wikipedia

Contents

• Popliteal artery and vein • Small saphenous vein • Common peroneal nerve • Tibial nerve • Posterior cutaneous nerve of the thigh • Genicular branch of the obturator nerve • Lymph nodes

A 76 year old man complains of symptoms of claudication. The decision is made to measure his ankle brachial pressure index. The signal from the dorsalis pedis artery is auscultated with a hand held doppler device. This vessel is the continuation of which of the following?

A. Posterior tibial artery

B. Anterior tibial artery

C. Peroneal artery

D. Popliteal artery

E. None of the above

The dorsalis pedis is a continuation of the anterior tibial artery.

Foot- anatomy Arches of the foot The foot is conventionally considered to have two arches.

Page 9: MRCS preparation emrcs questions Lowerlimb

• The longitudinal arch is higher on the medial than on the lateral side. The posterior part of the calcaneum forms a posterior pillar to support the arch. The lateral part of this structure passes via the cuboid bone and the lateral two metatarsal bones. The medial part of this structure is more important. The head of the talus marks the summit of this arch, located between the sustentaculum tali and the navicular bone. The anterior pillar of the medial arch is composed of the navicular bone, the three cuneiforms and the medial three metatarsal bones.

• The transverse arch is situated on the anterior part of the tarsus and the posterior part of the metatarsus. The cuneiforms and metatarsal bases narrow inferiorly, which contributes to the shape of the arch.

Intertarsal joints Sub talar joint Formed by the cylindrical facet on the lower surface of the

body of the talus and the posterior facet on the upper surface of the calcaneus. The facet on the talus is concave anteroposteriorly, the other is convex. The synovial cavity of this joint does not communicate with any other joint.

Talocalcaneonavicular joint

The anterior part of the socket is formed by the concave articular surface of the navicular bone, posteriorly by the upper surface of the sustentaculum tali. The talus sits within this socket

Calcaneocuboid joint Highest point in the lateral part of the longitudinal arch. The lower aspect of this joint is reinforced by the long plantar and plantar calcaneocuboid ligaments.

Transverse tarsal joint The talocalcaneonavicular joint and the calcaneocuboid joint extend accross the tarsus in an irregular transverse plane, between the talus and calcaneus behind and the navicular and cuboid bones in front. This plane is termed the transverse tarsal joint.

Cuneonavicular joint Formed between the convex anterior surface of the navicular bone and the concave surface of the the posterior ends of the three cuneiforms.

Intercuneiform joints Between the three cuneiform bones. Cuneocuboid joint Between the circular facets on the lateral cuneiform bone

and the cuboid. This joint contributes to the tarsal part of the transverse arch.

A detailed knowledge of the joints is not required for MRCS Part A. However, the contribution they play to the overall structure of the foot should be appreciated Ligaments of the ankle joint and foot

Page 10: MRCS preparation emrcs questions Lowerlimb

Image sourced from Wikipedia

Muscles of the foot Muscle Origin Insertion Nerve

supply Action

Abductor hallucis

Medial side of the calcaneus, flexor retinaculum, plantar aponeurosis

Medial side of the base of the proximal phalanx

Medial plantar nerve

Abducts the great toe

Flexor digitorum brevis

Medial process of the calcaneus, plantar eponeurosis.

Via 4 tendons into the middle phalanges of the lateral 4 toes.

Medial plantar nerve

Flexes all the joints of the lateral 4 toes except for the interphalangeal joint.

Abductor digit minimi

From the tubercle of the calcaneus and from the plantar aponeurosis

Together with flexor digit minimi brevis into the lateral side of the base of the proximal phalanx of the little toe

Lateral plantar nerve

Abducts the little toe at the metatarsophalangeal joint

Flexor hallucis brevis

From the medial side of the plantar surface of the cuboid bone, from the

Into the proximal phalanx of

Medial plantar nerve

Flexes the metatarsophalangeal joint of the great toe.

Page 11: MRCS preparation emrcs questions Lowerlimb

adjacent part of the lateral cuneiform bone and from the tendon of tibialis posterior.

the great toe, the tendon contains a sesamoid bone

Adductor hallucis

Arises from two heads. The oblique head arises from the sheath of the peroneus longus tendon, and from the plantar surfaces of the bases of the 2nd, 3rd and 4th metatarsal bones. The transverse head arises from the plantar surface of the lateral 4 metatarsophalangeal joints and from the deep transverse metatarsal ligament.

Lateral side of the base of the proximal phalanx of the great toe.

Lateral plantar nerve

Adducts the great toe towards the second toe. Helps maintain the transverse arch of the foot.

Extensor digitorum brevis

On the dorsal surface of the foot from the upper surface of the calcaneus and its associated fascia

Via four thin tendons which run forward and medially to be inserted into the medial four toes. The lateral three tendons join with hoods of extensor digitorum longus.

Deep peroneal

Extend the metatarsophalangeal joint of the medial four toes. It is unable to extend the interphalangeal joint without the assistance of the lumbrical muscles.

Detailed knowledge of the foot muscles are not needed for the MRCS part A Nerves in the foot Lateral plantar nerve Passes anterolaterally towards the base of the 5th metatarsal between flexor digitorum brevis and flexor accessorius. On the medial aspect of the lateral plantar artery. At the base of the 5th metatarsal it splits into superficial and deep branches. Medial plantar nerve Passes forwards with the medial plantar artery under the cover of the flexor retinaculum to the interval between abductor hallucis and flexor digitorum brevis on the sole of the foot.

Page 12: MRCS preparation emrcs questions Lowerlimb

Plantar arteries Arise under the cover of the flexor retinaculum, midway between the tip of the medial malleolus and the most prominent part of the medial side of the heel.

• Medial plantar artery. Passes forwards medial to medial plantar nerve in the space between abductor hallucis and flexor digitorum brevis.Ends by uniting with a branch of the 1st plantar metatarsal artery.

• Lateral plantar artery. Runs obliquely across the sole of the foot. It lies lateral to the lateral plantar nerve. At the base of the 5th metatarsal bone it arches medially across the foot on the metatarsals

Dorsalis pedis artery This vessel is a direct continuation of the anterior tibial artery. It commences on the front of the ankle joint and runs to the proximal end of the first metatarsal space. Here is gives off the arcuate artery and continues forwards as the first dorsal metatarsal artery. It is accompanied by two veins throughout its length. It is crossed by the extensor hallucis brevis A 67 year old man is due to undergo a revisional total hip replacement using a posterior approach. After dividing gluteus maximus in the line of its fibres there is brisk arterial bleeding. Which of the following vessels is likely to be responsible?

A. Profunda femoris artery

B. External iliac artery

C. Internal iliac artery

D. Obturator artery

E. Inferior gluteal artery

The inferior gluteal artery runs on the deep surface of the gluteus maximus muscle. It is a branch of the internal iliac artery. It is commonly divided during the posterior approach to the hip joint.

Hip joint

• Head of femur articulates with acetabulum of the pelvis • Both covered by articular hyaline cartilage • The acetabulum forms at the union of the ilium, pubis, and ischium • The triradiate cartilage (Y-shaped growth plate) separates the pelvic bones • The acetabulum holds the femoral head by the acetabular labrum • Normal angle between femoral head and femoral shaft is 130o

Ligaments

Page 13: MRCS preparation emrcs questions Lowerlimb

• Transverse ligament: joints anterior and posterior ends of the articular cartilage

• Head of femur ligament (ligamentum teres): acetabular notch to the fovea. Contains arterial supply to head of femur in children.

Image sourced from Wikipedia

Image sourced from Wikipedia

Extracapsular ligaments

Page 14: MRCS preparation emrcs questions Lowerlimb

• Iliofemoral ligament: inverted Y shape. Anterior iliac spine to the trochanteric line

• Pubofemoral ligament: acetabulum to lesser trochanter • Ischiofemoral ligament: posterior support. Ischium to greater trochanter.

Blood supply Medial circumflex femoral and lateral circumflex femoral arteries (Branches of profunda femoris) 2 anastomoses: Cruciate and the trochanteric anastomoses (provides most of the blood to the head of the femur) Hence the need for hemiarthroplasty when there is a displaced femoral head fracture. These anastomoses exist between the femoral artery or profunda femoris and the gluteal vessels. Which of the following nerves passes through the greater and lesser sciatic foramina?

A. Pudendal nerve

B. Sciatic nerve

C. Superior gluteal nerve

D. Inferior gluteal nerve

E. Posterior cutaneous nerve of the thigh

Structures passing through the lesser and greater sciatic foramina (medial to lateral): PIN

• Pudendal nerve • Internal pudendal artery • Nerve to obturator internus

Theme from 2010 Exam The pudendal nerve originates from the ventral rami of the second, third, and fourth sacral nerves (S2, S3, S4). It passes between the piriformis and coccygeus muscles and exits the pelvis through the the greater sciatic foramen. It crosses the spine of the ischium and reenters the pelvis through the lesser sciatic foramen. It passes through the pudendal canal. The pudendal nerve gives off the inferior rectal nerves. It terminates into 2 branches: perineal nerve, and the dorsal nerve of the penis or the dorsal nerve of the clitoris.

Greater sciatic foramen Contents Nerves • Sciatic Nerve

Page 15: MRCS preparation emrcs questions Lowerlimb

• Superior and Inferior Gluteal Nerves • Internal Pudendal Nerve • Posterior Femoral Cutaneous Nerve • Nerve to Quadratus Femoris • Nerve to Obturator internus

Vessels • Superior Gluteal Artery and vein • Inferior Gluteal Artery and vein • Internal Pudendal Artery and vein

Piriformis The piriformis is a landmark for identifying structures passing out of the sciatic notch

• Above piriformis: Superior gluteal vessels • Below piriformis: Inferior gluteal vessels, sciatic nerve (10% pass through it,

<1% above it), posterior cutaneous nerve of the thigh

Greater sciatic foramen boundaries Anterolaterally Greater sciatic notch of the ilium Posteromedially Sacrotuberous ligament Inferior Sacrospinous ligament and the ischial spine Superior Anterior sacroiliac ligament The greater sciatic foramen

Page 16: MRCS preparation emrcs questions Lowerlimb

Image sourced from Wikipedia

Structures passing between both foramina (Medial to lateral)

• Pudendal nerve • Internal pudendal artery • Nerve to obturator internus

Contents of the lesser sciatic foramen

• Tendon of the obturator internus • Pudendal nerve • Internal pudendal artery and vein • Nerve to the obturator internus

A 65 year old man with long standing atrial fibrillation develops an embolus to the lower leg. The decision is made to perform an embolectomy, utilising a trans popliteal approach. After incising the deep fascia, which of the following structures will the surgeons encounter first on exploring the central region of the popliteal fossa?

A. Popliteal vein

Page 17: MRCS preparation emrcs questions Lowerlimb

B. Common peroneal nerve

C. Popliteal artery

D. Tibial nerve

E. None of the above

Theme from 2009 Exam The tibial nerve lies superior to the vessels in the inferior aspect of the popliteal fossa. In the upper part of the fossa the tibial nerve lies lateral to the vessels, it then passes superficial to them to lie medially. The popliteal artery is the deepest structure in the popliteal fossa.

Popliteal fossa Boundaries of the popliteal fossa Laterally Biceps femoris above, lateral head of gastrocnemius and plantaris below Medially Semimembranosus and semitendinosus above, medial head of

gastrocnemius below Floor Popliteal surface of the femur, posterior ligament of knee joint and

popliteus muscle Roof Superficial and deep fascia Image showing the popliteal fossa

Image sourced from Wikipedia

Contents

• Popliteal artery and vein • Small saphenous vein • Common peroneal nerve • Tibial nerve

Page 18: MRCS preparation emrcs questions Lowerlimb

• Posterior cutaneous nerve of the thigh • Genicular branch of the obturator nerve • Lymph nodes

A 43 year old lady presents with varicose veins and undergoes a saphenofemoral disconnection, long saphenous vein stripping to the ankle and isolated hook phlebectomies. Post operatively she notices an area of numbness superior to her ankle. What is the most likely cause for this?

A. Sural nerve injury

B. Femoral nerve injury

C. Saphenous nerve injury

D. Common peroneal nerve injury

E. Superficial peroneal nerve injury

The sural nerve is related to the short saphenous vein. The saphenous nerve is related to the long saphenous vein below the knee and for this reason full length stripping of the vein is no longer advocated.

Saphenous vein Long saphenous vein This vein may be harvested for triple or quadruple bypass surgery

• Originates at the 1st digit where the dorsal vein merges with the dorsal venous arch of the foot

• Passes anterior to the medial malleolus and runs up the medial side of the leg • At the knee, it runs over the posterior border of the medial epicondyle of the

femur bone • Then passes laterally to lie on the anterior surface of the thigh before entering

an opening in the fascia lata called the saphenous opening • It joins with the femoral vein in the region of the femoral triangle at the

saphenofemoral junction

Tributaries

• Medial marginal • Superficial epigastric • Superficial iliac circumflex • Superficial external pudendal veins

Page 19: MRCS preparation emrcs questions Lowerlimb

Short saphenous vein

• Originates at the 5th digit where the dorsal vein merges with the dorsal venous arch of the foot, which attaches to the great saphenous vein.

• It passes around the lateral aspect of the foot (inferior and posterior to the lateral malleolus) and runs along the posterior aspect of the leg (with the sural nerve)

• Passes between the heads of the gastrocnemius muscle, and drains into the popliteal vein, approximately at or above the level of the knee joint.

A 34 year old man undergoes excision of a sarcoma from the right buttock. During the procedure the sciatic nerve is sacrificed. Which of the following will not occur as a result of this process?

A. Loss of extension at the knee joint

B. Foot drop

C. Inability to extend extensor hallucis longus

D. Loss of sensation to the posterior aspect of the thigh

E. Loss of sensation to the posterior aspect of the lower leg

Extension of the knee joint is caused by the obturator and femoral nerves.

Sciatic nerve Origin Spinal nerves L4 - S3 Articular Branches Hip joint Muscular branches in upper leg

• Semitendinosus • Semimembranosus • Biceps femoris • Part of adductor magnus

Cutaneous sensation • Posterior aspect of thigh • Gluteal region • Entire lower leg (except the medial aspect)

Terminates At the upper part of the popliteal fossa by dividing into the tibial and peroneal nerves

• The nerve to the short head of the biceps femoris comes from the common peroneal part of the sciatic and the other muscular branches arise from the tibial portion.

Page 20: MRCS preparation emrcs questions Lowerlimb

• The tibial nerve goes on to innervate all muscles of the foot except the extensor digitorum brevis (which is innervated by the common peroneal nerve).

• 24 year old lady is stabbed in the buttock. Following the injury the wound is sutured in the emergency department. Eight weeks later she attends the clinic, as she walks into the clinic room she has a waddling gait and difficulty with thigh abduction. On examination she has buttock muscle wasting. Which nerve has been injured?

A. Superior gluteal nerve

B. Obturator nerve

C. Sciatic nerve

D. Femoral nerve

E. Inferior gluteal nerve

• Theme from April 2012 Exam Damage to the superior gluteal nerve will result in a Trendelenberg gait.

• Trendelenberg test •

Injury or division of the superior gluteal nerve results in a motor deficit that consists of weakened abduction of the thigh by gluteus medius, a disabling gluteus medius limp and a compensatory list of the body weakened gluteal side. The compensation results in a gravitational shift so that the body is supported on the unaffected limb. When a person is asked to stand on one leg the gluteus medius usually contracts as soon as the contralateral leg leaves the floor, preventing the pelvis from dipping towards the unsupported side. When a person with paralysis of the superior gluteal nerve is asked to stand on one leg, the pelvis on the unsupported side descends, indicating that the gluteus medius on the affected side is weak or non functional ( a positive Trendelenberg test).

A 73 year old lady presents with symptoms of faecal incontinence. On examination she has weak anal sphincter muscles. What are the main nerve root values of the nerves supplying the external anal sphincter?

A. S2,3

B. L5, S1

C. S4,5

Page 21: MRCS preparation emrcs questions Lowerlimb

D. S5

E. S2,3,4

Theme from September 2011 Exam The external anal sphincter is innervated by the inferior rectal branch of the pudendal nerve, this has root values of S2, 3 and the perineal branch of S4.

Anal sphincter

• Internal anal sphincter composed of smooth muscle continuous with the circular muscle of the rectum. It surrounds the upper two- thirds of the anal canal and is supplied by sympathetic nerves.

• External anal sphincter is composed of striated muscle which surrounds the internal sphincter but extends more distally.

• The nerve supply of the external anal sphincter is from the inferior rectal branch of the pudendal nerve (S2 and S3) and the perineal branch of the S4 nerve roots.

Image showing relationship of internal and external anal sphincters

Image sourced from Wikipedia

A 72 year old man has a fall. He is found to have a fractured neck of femur and goes on to have a left hip hemiarthroplasty. Two months post operatively he is found to have an odd gait. When standing on his left leg his pelvis dips on the right side. There is no foot drop. What is the cause?

Page 22: MRCS preparation emrcs questions Lowerlimb

A. Sciatic nerve damage

B. L5 radiculopathy

C. Inferior gluteal nerve damage

D. Previous poliomyelitis

E. Superior gluteal nerve damage

Theme from 2010 Exam This patient has a trendelenburg gait caused by damage to the superior gluteal nerve causing weakness of the abductor muscles. Classically a patient is asked to stand on one leg and the pelvis dips on the opposite side. The absence of a foot drop excludes the possibility of polio or L5 radiculopathy.

Gluteal region Gluteal muscles

• Gluteus maximus: inserts to gluteal tuberosity of the femur and iliotibial tract • Gluteus medius: attach to lateral greater trochanter • Gluteus minimis: attach to anterior greater trochanter • All extend and abduct the hip

Deep lateral hip rotators

• Piriformis • Gemelli • Obturator internus • Quadratus femoris

Nerves Superior gluteal nerve (L5, S1) • Gluteus medius

• Gluteus minimis • Tensor fascia lata

Inferior gluteal nerve Gluteus maximus Damage to the superior gluteal nerve will result in the patient developing a Trendelenberg gait. Affected patients are unable to abduct the thigh at the hip joint. During the stance phase, the weakened abductor muscles allow the pelvis to tilt down on the opposite side. To compensate, the trunk lurches to the weakened side to attempt to maintain a level pelvis throughout the gait cycle. The pelvis sags on the opposite side of the lesioned superior gluteal nerve. Which of the following structures lies posterior to the femoral nerve in the femoral triangle?

Page 23: MRCS preparation emrcs questions Lowerlimb

A. Adductor longus

B. Pectineus

C. Psoas major

D. Iliacus

E. None of the above

The iliacus lies posterior to the femoral nerve in the femoral triangle. The femoral sheath lies anterior to the iliacus and pectineus muscles.

Femoral nerve

Root values L2, 3, 4

Innervates • Pectineus • Sartorius • Quadriceps femoris • Vastus lateralis/medialis/intermedius

Branches • Medial cutaneous nerve of thigh • Saphenous nerve • Intermediate cutaneous nerve of thigh

Path Penetrates psoas major and exits the pelvis by passing under the inguinal ligament to enter the femoral triangle, lateral to the femoral artery and vein.

Page 24: MRCS preparation emrcs questions Lowerlimb

Image sourced from Wikipedia

Mnemonic for femoral nerve supply (don't) M I S V Q Scan for PE M edial cutaneous nerve of the thigh I ntermediate cutaneous nerve of the thigh S aphenous nerve V astus Q uadriceps femoris S artorius PE ectineus Which of the following ligaments contains the artery supplying the head of femur in children?

A. Transverse ligament

Page 25: MRCS preparation emrcs questions Lowerlimb

B. Ligamentum teres

C. Iliofemoral ligament

D. Ischiofemoral ligament

E. Pubofemoral ligament

Theme from 2010 Exam

Hip joint

• Head of femur articulates with acetabulum of the pelvis • Both covered by articular hyaline cartilage • The acetabulum forms at the union of the ilium, pubis, and ischium • The triradiate cartilage (Y-shaped growth plate) separates the pelvic bones • The acetabulum holds the femoral head by the acetabular labrum • Normal angle between femoral head and femoral shaft is 130o

Ligaments

• Transverse ligament: joints anterior and posterior ends of the articular cartilage • Head of femur ligament (ligamentum teres): acetabular notch to the fovea. Contains

arterial supply to head of femur in children.

Page 26: MRCS preparation emrcs questions Lowerlimb

Image sourced from Wikipedia

Image sourced from Wikipedia

Extracapsular ligaments

• Iliofemoral ligament: inverted Y shape. Anterior iliac spine to the trochanteric line • Pubofemoral ligament: acetabulum to lesser trochanter • Ischiofemoral ligament: posterior support. Ischium to greater trochanter.

Blood supply Medial circumflex femoral and lateral circumflex femoral arteries (Branches of profunda femoris) 2 anastomoses: Cruciate and the trochanteric anastomoses (provides most of the blood to the head of the femur) Hence the need for hemiarthroplasty when there is a displaced femoral head fracture. These anastomoses exist between the femoral artery or profunda femoris and the gluteal vessels. A 68 year old man with critical limb ischaemia is undergoing a femoro-distal bypass graft. During mobilisation of the proximal part of the posterior tibial artery which of the following is at greatest risk of injury?

A. Tibial nerve

Page 27: MRCS preparation emrcs questions Lowerlimb

B. Sciatic nerve

C. Saphenous nerve

D. Common peroneal nerve

E. Medial superior genicular artery

The tibial nerve is closely related to the posterior tibial artery. The tibial nerve crosses the vessel posteriorly approximately 2.5cm distal to its origin. At its origin the nerve lies medial and then lateral after it crosses the vessel as described.

Posterior tibial artery

• Larger terminal branch of the popliteal artery • Terminates by dividing into the medial and lateral plantar arteries • Accompanied by two veins throughout its length • Position of the artery corresponds to a line drawn from the lower angle of the

popliteal fossa, at the level of the neck of the fibula, to a point midway between the medial malleolus and the most prominent part of the heel

Relations of the posterior tibial artery Proximal to distal Anteriorly Tibialis posterior

Flexor digitorum longus Posterior surface of tibia and ankle joint

Posterior Tibial nerve 2.5 cm distal to its origin Fascia overlying the deep muscular layer Proximal part covered by gastrocnemius and soleus Distal part covered by skin and fascia

Which of the following structures lies deepest in the popliteal fossa?

A. Popliteal artery

B. Popliteal vein

C. Tibial nerve

D. Common peroneal nerve

E. Popliteal lymph nodes

Theme from January 2012 exam From superficial to deep:

Page 28: MRCS preparation emrcs questions Lowerlimb

The common peroneal nerve exits the popliteal fossa along the medial border of the biceps tendon. Then the tibial nerve lies lateral to the popliteal vessels to pass posteriorly and then medially to them. The popliteal vein lies superficial to the popliteal artery, which is the deepest structure in the fossa.

Popliteal fossa Boundaries of the popliteal fossa Laterally Biceps femoris above, lateral head of gastrocnemius and plantaris below Medially Semimembranosus and semitendinosus above, medial head of

gastrocnemius below Floor Popliteal surface of the femur, posterior ligament of knee joint and

popliteus muscle Roof Superficial and deep fascia Image showing the popliteal fossa

Im An intravenous drug user develops a false aneurysm and requires emergency surgery. The procedure is difficult and the femoral nerve is inadvertently transected. Which of the following muscles is least likely to be affected as a result?

A. Sartorius

B. Vastus medialis

C. Pectineus

D. Quadriceps femoris

E. Adductor magnus

Page 29: MRCS preparation emrcs questions Lowerlimb

Mnemonic for femoral nerve supply (don't) M I S V Q Scan for PE M edial cutaneous nerve of the thigh I ntermediate cutaneous nerve of the thigh S aphenous nerve V astus Q uadriceps femoris S artorius PE ectineus

Adductor magnus is innervated by the obturator and sciatic nerve. The pectineus muscle is sometimes supplied by the obturator nerve but this is variable. Since the question states least likely, the correct answer is adductor magnus

Femoral nerve

Root values L2, 3, 4

Innervates • Pectineus • Sartorius • Quadriceps femoris • Vastus lateralis/medialis/intermedius

Branches • Medial cutaneous nerve of thigh • Saphenous nerve • Intermediate cutaneous nerve of thigh

Path Penetrates psoas major and exits the pelvis by passing under the inguinal ligament to enter the femoral triangle, lateral to the femoral artery and vein.

Page 30: MRCS preparation emrcs questions Lowerlimb

Image sourced from Wikipedia

Mnemonic for femoral nerve supply (don't) M I S V Q Scan for PE M edial cutaneous nerve of the thigh I ntermediate cutaneous nerve of the thigh S aphenous nerve V astus Q uadriceps femoris S artorius PE ectineus

Which of the following structures does not pass posteriorly to the medial malleolus?

A. Posterior tibial artery

B. Tibial nerve

Page 31: MRCS preparation emrcs questions Lowerlimb

C. Tibialis anterior tendon

D. Tendon of flexor digitorum longus

E. Tendon of flexor hallucis longus

Mnemonic for structures posterior to the medial malleolus: Tom Dick And Nervous Harry T ibialis posterior tendon flexor Digitorum longus A rtery N erve H allucis longus

Medial malleolus The following structures pass posterior to medial malleolus (in order):

• Tibialis posterior tendon • Flexor digitorum longus tendon • Posterior tibial artery • Tibial nerve • Tendon of flexor hallucis longus

A 44 year old man has a malignant melanoma and is undergoing a block dissection of the groin. The femoral triangle is being explored for intra operative bleeding. Which of the following forms the medial border of the femoral triangle?

A. Femoral artery

B. Biceps femoris

C. Adductor longus

D. Sartorius

E. Adductor magnus

Page 32: MRCS preparation emrcs questions Lowerlimb

Vastus medialis forms the lateral border of the adductor canal. The sartorius muscles forms the roof of the adductor canal.

Adductor longus forms the medial boundary of the femoral triangle (see below).

Femoral triangle anatomy Boundaries

Superiorly Inguinal ligament

Laterally Sartorius

Medially Adductor longus

Floor Iliopsoas, adductor longus and pectineus

Roof • Fascia lata and Superficial fascia • Superficial inguinal lymph nodes (palpable below the inguinal ligament) • Great saphenous vein

Page 33: MRCS preparation emrcs questions Lowerlimb

Image sourced from Wikipedia

Contents

• Femoral vein (medial to lateral) • Femoral artery-pulse palpated at the mid inguinal point • Femoral nerve • Deep and superficial inguinal lymph nodes • Lateral cutaneous nerve • Great saphenous vein • Femoral branch of the genitofemoral nerve

The foramen marking the termination of the adductor canal is located in which of the following?

A. Adductor longus

B. Adductor magnus

C. Adductor brevis

D. Sartorius

E. Semimembranosus

The foramen marking the distal limit of the adductor canal is contained within adductor magnus. The vessel passes through this region to enter the popliteal fossa.

Adductor canal

• Also called Hunter's or subsartorial canal

• Immediately distal to the apex of the femoral triangle, lying in the middle third of the thigh. Canal terminates at the adductor hiatus.

Borders Contents

Page 34: MRCS preparation emrcs questions Lowerlimb

Laterally Vastus medialis muscle Saphenous nerve

Posteriorly Adductor longus, adductor magnus Superficial femoral artery

Roof Sartorius Superficial femoral vein

In the image below the sartorius muscle is removed to expose the canal contents

Image sourced from Wikipedia

A 24 year old motor cyclist is involved in a road traffic accident. He suffers a tibial fracture which is treated with an intra medullary nail. Post operatively he develops a compartment syndrome. Surgical decompression of the anterior compartment will relieve pressure on all of the following muscles except?

A. Peroneus brevis

B. Peroneus tertius

Page 35: MRCS preparation emrcs questions Lowerlimb

C. Extensor digitorum longus

D. Tibialis anterior

E. None of the above

The anterior compartment contains: Tibialis anterior Extensor digitorum longus Peroneus tertius Extensor hallucis longus Anterior tibial artery All the muscles are innervated by the deep peroneal nerve.

Lower limb- Muscular compartments Anterior compartment Muscle Nerve Action Tibialis anterior Deep peroneal

nerve Dorsiflexes ankle joint, inverts foot

Extensor digitorum longus

Deep peroneal nerve

Extends lateral four toes, dorsiflexes ankle joint

Peroneus tertius Deep peroneal nerve

Dorsiflexes ankle, everts foot

Extensor hallucis longus

Deep peroneal nerve

Dorsiflexes ankle joint, extends big toe

Peroneal compartment Muscle Nerve Action Peroneus longus Superficial peroneal nerve Everts foot, assists in plantar flexion Peroneus brevis Superficial peroneal nerve Plantar flexes the ankle joint Superficial posterior compartment <muscle< b=""></muscle<>

Nerve Action

Gastrocnemius Tibial nerve

Plantar flexes the foot, may also flex the knee

Soleus Tibial nerve

Plantar flexor

Deep posterior compartment Muscle Nerve Action Flexor digitorum longus Tibial Flexes the lateral four toes Flexor hallucis longus Tibial Flexes the great toe Tibialis posterior Tibial Plantar flexor, inverts the foot

Page 36: MRCS preparation emrcs questions Lowerlimb

A 42 year old lady is reviewed in the outpatient clinic following a routine surgical procedure. She complains of diminished sensation at the lateral aspect of her foot. Which of the following nerves is likely to be affected?

A. Sural

B. Superficial peroneal

C. Deep peroneal

D. Medial plantar

E. Lateral Plantar

Theme from April 2012 Exam The sural nerve supplies the lateral aspect of the foot. It runs alongside the short saphenous vein and may be injured in short saphenous vein surgery.

Foot- Cutaneous sensation Region Nerve Lateral plantar Sural Dorsum (not 1st web space) Superficial peroneal 1st Web space Deep peroneal Extremities of toes Medial and lateral plantar nerves Proximal plantar Tibial Medial plantar Medial plantar nerve Lateral plantar Lateral plantar nerve

Page 37: MRCS preparation emrcs questions Lowerlimb

A sprinter attends A&E with severe leg pain. He had forgotten to warm up and ran a 100m sprint race. Towards the end of the race he experienced pain in the posterior aspect of his thigh. The pain worsens, localising to the lateral aspect of the knee. The sprinter is unable to flex the knee. What structure has been injured?

A. Anterior cruciate ligament

B. Posterior cruciate ligament

C. Semimembranosus tendon

Page 38: MRCS preparation emrcs questions Lowerlimb

D. Semiteninosus tendon

E. Biceps femoris tendon

Theme from 2009 Exam The biceps femoris is commonly injured in sports that require explosive bending of the knee as seen in sprinting, especially if the athlete has not warmed up first. Avulsion most commonly occurs where the long head attaches to the ischial tuberosity. Injuries to biceps femoris are more common than to the other hamstrings.

Biceps femoris The biceps femoris is one of the hamstring group of muscles located in the posterior upper thigh. It has two heads. Long head

Origin Ischial tuberosity

Insertion Fibular head

Action Knee flexion, lateral rotation tibia, extension hip

Innervation Tibial nerve (L5, S1, S2)

Arterial supply

Profunda femoris artery, inferior gluteal artery, and the superior muscular branches of popliteal artery

Image demonstrating the biceps femoris muscle, with the long head outlined

Page 39: MRCS preparation emrcs questions Lowerlimb

Image sourced from Wikipedia

Short head

Page 40: MRCS preparation emrcs questions Lowerlimb

Origin Lateral lip of linea aspera, lateral supracondylar ridge of femur

Insertion Fibular head

Action Knee flexion, lateral rotation tibia

Innervation Common peroneal nerve (L5, S1, S2)

Arterial supply

Profunda femoris artery, inferior gluteal artery, and the superior muscular branches of popliteal artery

he sciatic nerve lies deep to the following structures except:

A. Gluteus maximus

B. The femoral cutaneous nerve

C. Long head of biceps femoris

D. Gluteus medius

E. Branch of the inferior gluteal artery

The gluteus medius does not extend around to the sciatic nerve.

Sciatic nerve Origin Spinal nerves L4 - S3 Articular Branches Hip joint Muscular branches in upper leg

• Semitendinosus • Semimembranosus • Biceps femoris • Part of adductor magnus

Cutaneous sensation • Posterior aspect of thigh • Gluteal region • Entire lower leg (except the medial aspect)

Terminates At the upper part of the popliteal fossa by dividing into the tibial and peroneal nerves

• The nerve to the short head of the biceps femoris comes from the common peroneal part of the sciatic and the other muscular branches arise from the tibial portion.

Page 41: MRCS preparation emrcs questions Lowerlimb

• The tibial nerve goes on to innervate all muscles of the foot except the extensor digitorum brevis (which is innervated by the common peroneal nerve).

A 72 year old lady is suspected of having a femoral hernia. At which of the following sites is it most likely to be identifiable clinically?

A. Mid inguinal point

B. Above and medial to the pubic tubercle

C. Below and lateral to the pubic tubercle

D. Mid point of the inguinal ligament

E. 3 cm superomedially to the superficial inguinal ring

Femoral hernias exit the femoral canal below and lateral to the pubic tubercle. Femoral hernia occur mainly in women due to their difference in pelvic anatomy. They are at high risk of strangulation and therefore should be repaired.

Femoral canal The femoral canal lies at the medial aspect of the femoral sheath. The femoral sheath is a fascial tunnel containing both the femoral artery laterally and femoral vein medially. The canal lies medial to the vein. Borders of the femoral canal

Laterally Femoral vein

Medially Lacunar ligament

Anteriorly Inguinal ligament

Posteriorly Pectineal ligament

Image showing dissection of femoral canal

Page 42: MRCS preparation emrcs questions Lowerlimb

Image sourced from Wikipedia

Contents

• Lymphatic vessels • Cloquet's lymph node

Physiological significance Allows the femoral vein to expand to allow for increased venous return to the lower limbs. Pathological significance As a potential space, it is the site of femoral hernias. The relatively tight neck places these at high risk of strangulation. hich of the following represents the root values of the sciatic nerve?

A. L4 to S3

B. L1 to L4

Page 43: MRCS preparation emrcs questions Lowerlimb

C. L3 to S1

D. S1 to S4

E. L5 to S1

The sciatic nerve most commonly arises from L4 to S3.

Sciatic nerve Origin Spinal nerves L4 - S3 Articular Branches Hip joint Muscular branches in upper leg

• Semitendinosus • Semimembranosus • Biceps femoris • Part of adductor magnus

Cutaneous sensation • Posterior aspect of thigh • Gluteal region • Entire lower leg (except the medial aspect)

Terminates At the upper part of the popliteal fossa by dividing into the tibial and peroneal nerves

• The nerve to the short head of the biceps femoris comes from the common peroneal part of the sciatic and the other muscular branches arise from the tibial portion.

• The tibial nerve goes on to innervate all muscles of the foot except the extensor digitorum brevis (which is innervated by the common peroneal nerve).

• he common peroneal nerve, or its branches, supply the following muscles except:

A. Peroneus longus

B. Tibialis anterior

C. Extensor hallucis longus

D. Flexor digitorum brevis

E. Extensor digitorum longus

• Flexor digitorum is supplied by the tibial nerve.

• Common peroneal nerve

Page 44: MRCS preparation emrcs questions Lowerlimb

• Derived from the dorsal divisions of the sacral plexus (L4, L5, S1 and S2). This nerve supplies the skin and fascia of the anterolateral surface of the leg and the dorsum of the foot. It also innervates the muscles of the anterior and peroneal compartments of the leg, extensor digitorum brevis as well as the knee, ankle and foot joints. It is laterally placed within the sciatic nerve. From the bifurcation of the sciatic nerve it passes inferolaterally in the lateral and proximal part of the popliteal fossa, under the cover of biceps femoris and its tendon. To reach the posterior aspect of the fibular head. It ends by dividing into the deep and superficial peroneal nerves at the point where it winds around the lateral surface of the neck of the fibula in the body of peroneus longus, approximately 2cm distal to the apex of the head of the fibula. It is palpable posterior to the head of the fibula. Branches

In the thigh Nerve to the short head of biceps Articular branch (knee)

In the popliteal fossa Lateral cutaneous nerve of the calf Neck of fibula Superficial and deep peroneal nerves An 83 year old lady presents with a femoral hernia and undergoes a femoral hernia repair. Which of the following forms the posterior wall of the femoral canal?

A. Pectineal ligament

B. Lacunar ligament

C. Inguinal ligament

D. Adductor longus

E. Sartorius

Femoral canal The femoral canal lies at the medial aspect of the femoral sheath. The femoral sheath is a fascial tunnel containing both the femoral artery laterally and femoral vein medially. The canal lies medial to the vein.

Page 45: MRCS preparation emrcs questions Lowerlimb

Borders of the femoral canal

Laterally Femoral vein

Medially Lacunar ligament

Anteriorly Inguinal ligament

Posteriorly Pectineal ligament

Image showing dissection of femoral canal

Image sourced from Wikipedia

Contents

• Lymphatic vessels • Cloquet's lymph node

Page 46: MRCS preparation emrcs questions Lowerlimb

Physiological significance Allows the femoral vein to expand to allow for increased venous return to the lower limbs. Pathological significance As a potential space, it is the site of femoral hernias. The relatively tight neck places these at high risk of strangulation. hich of the following structures does not pass behind the lateral malleolus?

A. Peroneus brevis tendon

B. Sural nerve

C. Short saphenous vein

D. Peroneus longus tendon

E. Tibialis anterior tendon

Tibialis anterior tendon passes at the medial malleolus.

Lateral malleolus Structures posterior to the lateral malleolus and superficial to superior peroneal retinaculum

• Sural nerve • Short saphenous vein

Structures posterior to the lateral malleolus and deep to superior peroneal retinaculum

• Peroneus longus tendon • Peroneus brevis tendon

The calcaneofibular ligament is attached at the lateral malleolus Theme: Nerve lesions

A. Iliohypogastric nerve B. Ilioinguinal nerve C. Lateral cutaneous nerve of the thigh D. Femoral nerve E. Saphenous nerve F. Genitofemoral nerve

Page 47: MRCS preparation emrcs questions Lowerlimb

Please select the most likely nerve implicated in the situation described. Each option may be used once, more than once or not at all.

14. A 42 year old woman complains of a burning pain of her anterior thigh which worsens on walking. There is a positive tinel sign over the inguinal ligament.

You answered Genitofemoral nerve The correct answer is Lateral cutaneous nerve of the thigh The lateral cutaneous nerve supplies sensation to the anterior and lateral aspect of the thigh. Entrapment is commonly due to intra and extra pelvic causes. Treatment involves local anaesthetic injections.

15. A 29 year old woman has had a Pfannenstiel incision. She has pain over the inguinal ligament which radiates to the lower abdomen. There is tenderness when the inguinal canal is compressed.

You answered Iliohypogastric nerve The correct answer is Ilioinguinal nerve

16. A 22 year man is shot in the groin. On examination he has weak hip flexion, weak knee extension, and impaired quadriceps tendon reflex, as well as sensory deficit in the anteromedial aspect of the thigh.

Femoral nerve This is a classical description of a femoral nerve injury.

Nerve lesions during surgery A variety of different procedures carry the risk of iatrogenic nerve injury. These are important not only from the patients perspective but also from a medicolegal standpoint. The following operations and their associated nerve lesions are listed here:

• Posterior triangle lymph node biopsy and accessory nerve lesion. • Lloyd Davies stirrups and common peroneal nerve. • Thyroidectomy and laryngeal nerve. • Anterior resection of rectum and hypogastric autonomic nerves.

Page 48: MRCS preparation emrcs questions Lowerlimb

• Axillary node clearance; long thoracic nerve, thoracodorsal nerve and intercostobrachial nerve.

• Inguinal hernia surgery and ilioinguinal nerve. • Varicose vein surgery- sural and saphenous nerves. • Posterior approach to the hip and sciatic nerve. • Carotid endarterectomy and hypoglossal nerve.

There are many more, with sound anatomical understanding of the commonly performed procedures the incidence of nerve lesions can be minimised. They commonly occur when surgeons operate in an unfamiliar tissue plane or by blind placement of haemostats (not recommended). 23 year old man is stabbed in the groin, several structures are injured and the adductor longus muscle has been lacerated. Which of the following nerves is responsible for the innervation of adductor longus?

A. Femoral nerve

B. Obturator nerve

C. Sciatic nerve

D. Common peroneal nerve

E. Ilioinguinal nerve

The adductors are innervated by the obturator nerve

Adductor longus

Origin Anterior body of pubis

Insertion Middle third of linea aspera

Action Adducts and flexes the thigh, medially rotate the hip

Innervation Anterior division of obturator nerve (L2, L3, L4)

The schematic image below demonstrates the relationship of the adductor muscles

Page 49: MRCS preparation emrcs questions Lowerlimb

Image sourced from Wikipedia

hich of the following muscles does not recieve any innervation from the sciatic nerve?

A. Semimembranosus

B. Quadriceps femoris

C. Biceps femoris

D. Semitendinosus

E. Adductor magnus

The sciatic nerve is traditionally viewed as being a nerve of the posterior compartment. It is known to contribute to the innervation of adductor magnus (although the main innervation to this muscle is from the obturator nerve). The quadriceps femoris is nearly always innervated by the femoral nerve.

Sciatic nerve Origin Spinal nerves L4 - S3 Articular Branches Hip joint

Page 50: MRCS preparation emrcs questions Lowerlimb

Muscular branches in upper leg

• Semitendinosus • Semimembranosus • Biceps femoris • Part of adductor magnus

Cutaneous sensation • Posterior aspect of thigh • Gluteal region • Entire lower leg (except the medial aspect)

Terminates At the upper part of the popliteal fossa by dividing into the tibial and peroneal nerves

• The nerve to the short head of the biceps femoris comes from the common peroneal part of the sciatic and the other muscular branches arise from the tibial portion.

• The tibial nerve goes on to innervate all muscles of the foot except the extensor digitorum brevis (which is innervated by the common peroneal nerve).

The following are true of the femoral nerve except:

A. It is derived from L2, L3 and L4 nerve roots

B. It supplies sartorius

C. It supplies quadriceps femoris

D. It gives cutaneous innervations via the saphenous nerve

E. It supplies adductor longus

Adductor longus is supplied by the obturator nerve.

Femoral nerve

Root values L2, 3, 4

Innervates • Pectineus • Sartorius • Quadriceps femoris

Page 51: MRCS preparation emrcs questions Lowerlimb

• Vastus lateralis/medialis/intermedius

Branches • Medial cutaneous nerve of thigh • Saphenous nerve • Intermediate cutaneous nerve of thigh

Path Penetrates psoas major and exits the pelvis by passing under the inguinal ligament to enter the femoral triangle, lateral to the femoral artery and vein.

Image sourced from Wikipedia

Mnemonic for femoral nerve supply (don't) M I S V Q Scan for PE M edial cutaneous nerve of the thigh I ntermediate cutaneous nerve of the thigh S aphenous nerve V astus

Page 52: MRCS preparation emrcs questions Lowerlimb

Q uadriceps femoris S artorius PE ectineus

Which of the following is true in relation to the sartorius muscle?

A. Innervated by the deep branch of the femoral nerve

B. Inserts at the fibula

C. It is the shortest muscle in the body

D. Forms the Pes anserinus with Gracilis and semitendinous muscle

E. Causes extension of the knee

It is innervated by the superficial branch of the femoral nerve. It is a component of the pes anserinus.

Sartorius

• Longest strap muscle in the body • Most superficial muscle in the anterior compartment of the thigh

Origin Anterior superior iliac spine Insertion Medial surface of the of the body of the tibia (upper part). It inserts

anterior to gracilis and semitendinosus Nerve Supply

Femoral nerve (L2,3)

Action • Flexor of the hip and knee, slight abducts the thigh and rotates it laterally

• It assists with medial rotation of the tibia on the femur. For example it would play a pivotal role in placing the right heel onto the left knee ( and vice versa)

Important relations

The middle third of this muscle, and its strong underlying fascia forms the roof of the adductor canal , in which lie the femoral vessels, the saphenous nerve and the nerve to vastus medialis.

heme: Nerve lesions

A. Sciatic nerve

Page 53: MRCS preparation emrcs questions Lowerlimb

B. Peroneal nerve C. Tibial Nerve D. Obturator nerve E. Ilioinguinal nerve F. Femoral nerve G. None of the above Please select the most likely nerve injury for the scenario given. Each option may be used once, more than once or not at all

27. A 56 year old man undergoes a low anterior resection with legs in the Lloyd-Davies position. Post operatively he complains of foot drop.

Peroneal nerve Positioning legs in Lloyd- Davies stirrups can carry the risk of peroneal nerve neuropraxia if not done carefully.

28. A 23 year old man complains of severe groin pain several weeks after a difficult inguinal hernia repair.

Ilioinguinal nerve The ilioinguinal nerve may have been entrapped in the mesh causing a neuroma.

29. A 72 year old man develops a foot drop after a revision total hip replacement.

Sciatic nerve This may be done by a number of approaches, in this scenario a posterior approach is the most likely culprit.

Lower limb- Muscular compartments Anterior compartment Muscle Nerve Action Tibialis anterior Deep peroneal

nerve Dorsiflexes ankle joint, inverts foot

Extensor digitorum longus

Deep peroneal nerve

Extends lateral four toes, dorsiflexes ankle joint

Peroneus tertius Deep peroneal nerve

Dorsiflexes ankle, everts foot

Page 54: MRCS preparation emrcs questions Lowerlimb

Extensor hallucis longus

Deep peroneal nerve

Dorsiflexes ankle joint, extends big toe

Peroneal compartment Muscle Nerve Action Peroneus longus Superficial peroneal nerve Everts foot, assists in plantar flexion Peroneus brevis Superficial peroneal nerve Plantar flexes the ankle joint Superficial posterior compartment <muscle< b=""></muscle<>

Nerve Action

Gastrocnemius Tibial nerve

Plantar flexes the foot, may also flex the knee

Soleus Tibial nerve

Plantar flexor

Deep posterior compartment Muscle Nerve Action Flexor digitorum longus Tibial Flexes the lateral four toes Flexor hallucis longus Tibial Flexes the great toe Tibialis posterior Tibial Plantar flexor, inverts the foot Which muscle is supplied by the superficial peroneal nerve?

A. Peroneus tertius

B. Sartorius

C. Adductor magnus

D. Peroneus brevis

E. Gracilis

Superficial peroneal nerve Supplies

• Lateral compartment of leg: peroneus longus, peroneus brevis (action: eversion and plantar flexion)

• Sensation over dorsum of the foot (except the first web space, which is innervated by the deep peroneal nerve)

Path

Page 55: MRCS preparation emrcs questions Lowerlimb

• Passes between peroneus longus and peroneus brevis along the length of the proximal one third of the fibula

• 10-12 cm above the tip of the lateral malleolus, the superficial peroneal nerve pierces the fascia

• 6-7 cm distal to the fibula, the superficial peroneal nerve bifurcates into intermediate and medial dorsal cutaneous nerves

Which of the following structures separates the posterior cruciate ligament from the popliteal artery?

A. Oblique popliteal ligament

B. Transverse ligament

C. Popliteus tendon

D. Biceps femoris

E. Semitendinosus

The posterior cruciate ligament is separated from the popliteal vessels at its origin by the oblique popliteal ligament. The transverse ligament is located anteriorly.

Knee joint The knee joint is a synovial joint, the largest and most complicated. It consists of two condylar joints between the femure and tibia and a sellar joint between the patella and the femur. The tibiofemoral articular surfaces are incongruent, however, this is improved by the presence of the menisci. The degree of congruence is related to the anatomical position of the knee joint and is greatest in full extension. Knee joint compartments

Tibiofemoral • Comprise of the patella/femur joint, lateral and medial compartments (between femur condyles and tibia)

• Synovial membrane and cruciate ligaments partially separate the medial and lateral compartments

Patellofemoral • Ligamentum patellae • Actions: provides joint stability in full extension

Fibrous capsule

Page 56: MRCS preparation emrcs questions Lowerlimb

The capsule of the knee joint is a complex, composite structure with contributions from adjacent tendons.

Anterior fibres

The capsule does not pass proximal to the patella. It blends with the tendinous expansions of vastus medialis and lateralis

Posterior fibres

These fibres are vertical and run from the posterior surface of the femoral condyles to the posterior aspect of the tibial condyle

Medial fibres

Attach to the femoral and tibial condyles beyond their articular margins, blending with the tibial collateral ligament

Lateral fibres

Attach to the femur superior to popliteus, pass over its tendon to head of fibula and tibial condyle

Bursae

Anterior • Subcutaneous prepatellar bursa; between patella and skin • Deep infrapatellar bursa; between tibia and patellar ligament • Subcutaneous infrapatellar bursa; between distal tibial tuberosity and skin

Laterally • Bursa between lateral head of gastrocnemius and joint capsule • Bursa between fibular collateral ligament and tendon of biceps femoris • Bursa between fibular collateral ligament and tendon of popliteus

Medially • Bursa between medial head of gastrocnemius and the fibrous capsule • Bursa between tibial collateral ligament and tendons of sartorius, gracilis

and semitendinosus • Bursa between the tendon of semimembranosus and medial tibial condyle

and medial head of gastrocnemius

Posterior Highly variable and inconsistent

Ligaments

Medial collateral ligament

Medial epicondyle femur to medial tibial condyle: valgus stability

Lateral collateral ligament

Lateral epicondyle femur to fibula head: varus stability

Anterior cruciate ligament

Anterior tibia to lateral intercondylar notch femur: prevents tibia sliding anteriorly

Posterior cruciate Posterior tibia to medial intercondylar notch femur: prevents tibia

Page 57: MRCS preparation emrcs questions Lowerlimb

ligament sliding posteriorly

Patellar ligament Central band of the tendon of quadriceps femoris, extends from patella to tibial tuberosity

Image sourced from Wikipedia

Page 58: MRCS preparation emrcs questions Lowerlimb

Image sourced from Wikipedia

Menisci Medial and lateral menisci compensate for the incongruence of the femoral and tibial condyles. Composed of fibrous tissue. Medial meniscus is attached to the tibial collateral ligament. Lateral meniscus is attached to the loose fibres at the lateral edge of the joint and is separate from the fibular collateral ligament. The lateral meniscus is crossed by the popliteus tendon. Nerve supply The knee joint is supplied by the femoral, tibial and common peroneal divisions of the sciatic and by a branch from the obturator nerve. Hip pathology pain may be referred to the knee. Blood supply Genicular branches of the femoral artery, popliteal and anterior tibial arteries all supply the knee joint.

A 25 year old man undergoes an excision of a pelvic chondrosarcoma, during the operation the obturator nerve is sacrificed. Which of the following muscles is least likely to be affected as a result?

Page 59: MRCS preparation emrcs questions Lowerlimb

A. Adductor longus

B. Pectineus

C. Adductor magnus

D. Sartorius

E. Gracilis

Sartorius is supplied by the femoral nerve. In approximately 20% of the population, pectineus is supplied by the accessory obturator nerve.

Obturator nerve The obturator nerve arises from L2, L3 and L4 by branches from the ventral divisions of each of these nerve roots. L3 forms the main contribution and the second lumbar branch is occasionally absent. These branches unite in the substance of psoas major, descending vertically in its posterior part to emerge from its medial border at the lateral margin of the sacrum. It then crosses the sacroiliac joint to enter the lesser pelvis, it descends on obturator internus to enter the obturator groove. In the lesser pelvis the nerve lies lateral to the internal iliac vessels and ureter, and is joined by the obturator vessels lateral to the ovary or ductus deferens. Supplies

• Medial compartment of thigh • Muscles supplied: external obturator, adductor longus, adductor brevis, adductor

magnus (not the lower part-sciatic nerve), gracilis • The cutaneous branch is often absent. When present, it passes between gracilis and

adductor longus near the middle part of the thigh, and supplies the skin and fascia of the distal two thirds of the medial aspect.

Obturator canal

• Connects the pelvis and thigh: contains the obturator artery, vein, nerve which divides into anterior and posterior branches.

Cadaveric cross section demonstrating relationships of the obturator nerve

Page 60: MRCS preparation emrcs questions Lowerlimb

Image sourced from Wikipedia

Which nerve supplies the 1st web space of the foot?

A. Popliteal nerve

B. Superficial peroneal nerve

C. Deep peroneal nerve

D. Tibial nerve

E. Saphenous nerve

The first web space is innervated by the deep peroneal nerve. See diagram below:

Page 61: MRCS preparation emrcs questions Lowerlimb

Deep peroneal nerve Origin From the common peroneal nerve, at the lateral aspect of the

fibula, deep to peroneus longus Nerve root values L4, L5, S1, S2 Course and relation

• Pierces the anterior intermuscular septum to enter the anterior compartment of the lower leg

• Passes anteriorly down to the ankle joint, midway between the two malleoli

Terminates In the dorsum of the foot Muscles innervated

• Tibialis anterior • Extensor hallucis longus • Extensor digitorum longus • Peroneus tertius

Page 62: MRCS preparation emrcs questions Lowerlimb

• Extensor digitorum brevis

Cutaneous innervation

Web space of the first and second toes

Actions • Dorsiflexion of ankle joint • Extension of all toes (extensor hallucis longus and extensor

digitorum longus) • Eversion of the foot

After its bifurcation past the ankle joint, the lateral branch of the deep peroneal nerve innervates the extensor digitorum brevis and the extensor hallucis brevis The medial branch supplies the web space between the first and second digits.

Which of the following structures does not pass behind the piriformis muscle in the greater sciatic foramen?

A. Sciatic nerve

B. Posterior cutaneous nerve of the thigh

C. Inferior gluteal artery

D. Obturator nerve

E. None of the above

The obturator nerve does not pass through the greater sciatic foramen.

Greater sciatic foramen Contents

Nerves • Sciatic Nerve • Superior and Inferior Gluteal Nerves • Internal Pudendal Nerve • Posterior Femoral Cutaneous Nerve • Nerve to Quadratus Femoris • Nerve to Obturator internus

Vessels • Superior Gluteal Artery and vein • Inferior Gluteal Artery and vein

Page 63: MRCS preparation emrcs questions Lowerlimb

• Internal Pudendal Artery and vein

Piriformis The piriformis is a landmark for identifying structures passing out of the sciatic notch

• Above piriformis: Superior gluteal vessels • Below piriformis: Inferior gluteal vessels, sciatic nerve (10% pass through it, <1%

above it), posterior cutaneous nerve of the thigh

Greater sciatic foramen boundaries

Anterolaterally Greater sciatic notch of the ilium

Posteromedially Sacrotuberous ligament

Inferior Sacrospinous ligament and the ischial spine

Superior Anterior sacroiliac ligament

The greater sciatic foramen

Page 64: MRCS preparation emrcs questions Lowerlimb

Image sourced from Wikipedia

Structures passing between both foramina (Medial to lateral)

• Pudendal nerve • Internal pudendal artery • Nerve to obturator internus

Contents of the lesser sciatic foramen

• Tendon of the obturator internus • Pudendal nerve • Internal pudendal artery and vein • Nerve to the obturator internus

A 78 year old lady falls over in her nursing home and sustains a displaced intracapsular fracture of the femoral neck. A decision is made to perform a hemi arthroplasty through a lateral approach. Which of the following vessels will be divided to facilitate access?

Page 65: MRCS preparation emrcs questions Lowerlimb

A. Saphenous vein

B. Superior gluteal artery

C. Superficial circumflex iliac artery

D. Profunda femoris artery

E. Transverse branch of the lateral circumflex artery

During the Hardinge style lateral approach the transverse branch of the lateral circumflex artery is divided to gain access. The vessels and its branches are illustrated below:

Page 66: MRCS preparation emrcs questions Lowerlimb

Image sourced from Wikipedia

Hip joint

• Head of femur articulates with acetabulum of the pelvis • Both covered by articular hyaline cartilage • The acetabulum forms at the union of the ilium, pubis, and ischium • The triradiate cartilage (Y-shaped growth plate) separates the pelvic bones • The acetabulum holds the femoral head by the acetabular labrum • Normal angle between femoral head and femoral shaft is 130o

Ligaments

Page 67: MRCS preparation emrcs questions Lowerlimb

• Transverse ligament: joints anterior and posterior ends of the articular cartilage • Head of femur ligament (ligamentum teres): acetabular notch to the fovea. Contains

arterial supply to head of femur in children.

Image sourced from Wikipedia

Image sourced from Wikipedia

Page 68: MRCS preparation emrcs questions Lowerlimb

Extracapsular ligaments

• Iliofemoral ligament: inverted Y shape. Anterior iliac spine to the trochanteric line • Pubofemoral ligament: acetabulum to lesser trochanter • Ischiofemoral ligament: posterior support. Ischium to greater trochanter.

Blood supply Medial circumflex femoral and lateral circumflex femoral arteries (Branches of profunda femoris) 2 anastomoses: Cruciate and the trochanteric anastomoses (provides most of the blood to the head of the femur) Hence the need for hemiarthroplasty when there is a displaced femoral head fracture. These anastomoses exist between the femoral artery or profunda femoris and the gluteal vessels. A 72 year old lady with osteoporosis falls and sustains an intracapsular femoral neck fracture. The fracture is completely displaced. Which of the following vessels is the main contributor to the arterial supply of the femoral head?

A. Deep external pudendal artery

B. Superficial femoral artery

C. External iliac artery

D. Circumflex femoral arteries

E. Superficial external pudendal artery

The vessels which form the anastomoses around the femoral head are derived from the medial and lateral circumflex femoral arteries. These are usually derived from the profunda femoris artery.

Hip joint

• Head of femur articulates with acetabulum of the pelvis • Both covered by articular hyaline cartilage • The acetabulum forms at the union of the ilium, pubis, and ischium • The triradiate cartilage (Y-shaped growth plate) separates the pelvic bones • The acetabulum holds the femoral head by the acetabular labrum

Page 69: MRCS preparation emrcs questions Lowerlimb

• Normal angle between femoral head and femoral shaft is 130o

Ligaments

• Transverse ligament: joints anterior and posterior ends of the articular cartilage • Head of femur ligament (ligamentum teres): acetabular notch to the fovea. Contains

arterial supply to head of femur in children.

Image sourced from Wikipedia

Page 70: MRCS preparation emrcs questions Lowerlimb

Image sourced from Wikipedia

Extracapsular ligaments

• Iliofemoral ligament: inverted Y shape. Anterior iliac spine to the trochanteric line • Pubofemoral ligament: acetabulum to lesser trochanter • Ischiofemoral ligament: posterior support. Ischium to greater trochanter.

Blood supply Medial circumflex femoral and lateral circumflex femoral arteries (Branches of profunda femoris) 2 anastomoses: Cruciate and the trochanteric anastomoses (provides most of the blood to the head of the femur) Hence the need for hemiarthroplasty when there is a displaced femoral head fracture. These anastomoses exist between the femoral artery or profunda femoris and the gluteal vessels. The following statements relating to the ankle joint are true except?

A. Three groups of ligaments provide mechanical stability

B. The sural nerve lies medial to the Achilles tendon at its point of insertion

C. Eversion of the foot occurs at the sub talar joint

Page 71: MRCS preparation emrcs questions Lowerlimb

D. The flexor hallucis longus tendon is the most posterior structure at the medial malleolus

E. The saphenous nerve crosses the ankle joint.

The sural nerve lies behind the distal fibula. Inversion and eversion are sub talar movements. The structures passing behind the medial malleolus from anterior to posterior include: tibialis posterior, flexor digitorum longus, posterior tibia vein, posterior tibial artery, nerve, flexor hallucis longus.

Ankle joint The ankle joint is a synovial joint composed of the tibia and fibula superiorly and the talus inferiorly. Ligaments of the ankle joint

• Deltoid ligament (medially) • Lateral collateral ligament • Talofibular ligaments (both anteriorly and posteriorly)

The calcaneofibular ligament is separate from the fibrous capsule of the joint. The two talofibular ligaments are fused with it. The components of the syndesmosis are

• Antero-inferior talofibular ligament • Postero-inferior talofibular ligament • Inferior transverse talofibular ligament • Interosseous ligament

Movements at the ankle joint

• Plantar flexion (55 degrees) • Dorsiflexion (35 degrees) • Inversion and eversion movements occur at the level of the sub talar joint

Nerve supply Branches of deep peroneal and tibial nerves. References Golano P et al. Anatomy of the ankle ligaments: a pictorial essay. Knee Surg Sports Traumatol Arthrosc. 2010 May;18(5):557-69 A 19 year old man is playing rugby when he suddenly notices a severe pain at the posterolateral aspect of his right thigh. Which of the following muscle groups is most likely to have been injured?

Page 72: MRCS preparation emrcs questions Lowerlimb

A. Semimembranosus

B. Semitendinosus

C. Long head of biceps femoris

D. Gastrocnemius

E. Soleus

Theme from April 2012 Exam The biceps femoris is the laterally located hamstring muscle. The semitendinosus and semimembranosus are located medially. Rupture of gastrocnemius and soleus may occur but is less common.

Biceps femoris The biceps femoris is one of the hamstring group of muscles located in the posterior upper thigh. It has two heads. Long head

Origin Ischial tuberosity

Insertion Fibular head

Action Knee flexion, lateral rotation tibia, extension hip

Innervation Tibial nerve (L5, S1, S2)

Arterial supply

Profunda femoris artery, inferior gluteal artery, and the superior muscular branches of popliteal artery

Image demonstrating the biceps femoris muscle, with the long head outlined

Page 73: MRCS preparation emrcs questions Lowerlimb

Image sourced from Wikipedia

Short head

Page 74: MRCS preparation emrcs questions Lowerlimb

Origin Lateral lip of linea aspera, lateral supracondylar ridge of femur

Insertion Fibular head

Action Knee flexion, lateral rotation tibia

Innervation Common peroneal nerve (L5, S1, S2)

Arterial supply

Profunda femoris artery, inferior gluteal artery, and the superior muscular branches of popliteal artery

A 22 year old man is involved in a fight and is stabbed in the posterior aspect of his right leg. The knife passes into the popliteal fossa. He sustains an injury to his tibial nerve. Which of the following muscles is least likely to be compromised as a result?

A. Tibialis posterior

B. Flexor hallucis longus

C. Flexor digitorum brevis

D. Soleus

E. Peroneus tertius

Peroneus tertius is innervated by the deep peroneal nerve.

Tibial nerve Begins at the upper border of the popliteal fossa and is a branch of the sciatic nerve.

• Root values: L4, L5, S1, S2, S3

Muscles innervated

• Popliteus • Gastrocnemius • Soleus • Plantaris • Tibialis posterior • Flexor hallucis longus • Flexor digitorum brevis

Terminates by dividing into the medial and lateral plantar nerves.

Rate question: 1

Page 75: MRCS preparation emrcs questions Lowerlimb

2 3 4 5

At which of the following anatomical locations does the common peroneal nerve bifurcate into the superficial and deep peroneal nerves?

A. Immediately anterior to the linea aspera

B. At the lateral aspect of the neck of the fibula

C. Within the substance of tibialis anterior muscle

D. At the inferomedial aspect of the popliteal fossa

E. Under the medial head of gastrocnemius

The common peroneal nerve bifurcates at the neck of the fibula (where it is most likely to be injured).

Common peroneal nerve Derived from the dorsal divisions of the sacral plexus (L4, L5, S1 and S2). This nerve supplies the skin and fascia of the anterolateral surface of the leg and the dorsum of the foot. It also innervates the muscles of the anterior and peroneal compartments of the leg, extensor digitorum brevis as well as the knee, ankle and foot joints. It is laterally placed within the sciatic nerve. From the bifurcation of the sciatic nerve it passes inferolaterally in the lateral and proximal part of the popliteal fossa, under the cover of biceps femoris and its tendon. To reach the posterior aspect of the fibular head. It ends by dividing into the deep and superficial peroneal nerves at the point where it winds around the lateral surface of the neck of the fibula in the body of peroneus longus, approximately 2cm distal to the apex of the head of the fibula. It is palpable posterior to the head of the fibula. Branches In the thigh Nerve to the short head of biceps

Articular branch (knee) In the popliteal fossa Lateral cutaneous nerve of the calf Neck of fibula Superficial and deep peroneal nerves A 48 year old motor cyclist sustains a complex lower limb fracture in a motor accident. For a time the popliteal artery is occluded and eventually repaired. Subsequently he develops a compartment syndrome and the anterior and superficial posterior compartments of the lower leg are decompressed. Unfortunately, the

Page 76: MRCS preparation emrcs questions Lowerlimb

operating surgeon neglects to decompress the deep posterior compartment. Which of the following muscles is least likely to be affected as a result?

A. Flexor digitorum longus

B. Plantaris

C. Tibialis posterior

D. Flexor hallucis longus

E. None of the above

Muscles of the deep posterior compartment:

• Tibialis posterior • Flexor hallucis longus • Flexor digitorum longus • Popliteus

The plantaris muscle lies within the superficial posterior compartment of the lower leg.

Lower limb- Muscular compartments Anterior compartment Muscle Nerve Action Tibialis anterior Deep peroneal

nerve Dorsiflexes ankle joint, inverts foot

Extensor digitorum longus

Deep peroneal nerve

Extends lateral four toes, dorsiflexes ankle joint

Peroneus tertius Deep peroneal nerve

Dorsiflexes ankle, everts foot

Extensor hallucis longus

Deep peroneal nerve

Dorsiflexes ankle joint, extends big toe

Peroneal compartment Muscle Nerve Action Peroneus longus Superficial peroneal nerve Everts foot, assists in plantar flexion Peroneus brevis Superficial peroneal nerve Plantar flexes the ankle joint Superficial posterior compartment <muscle< b=""></muscle<>

Nerve Action

Gastrocnemius Tibial Plantar flexes the foot, may also flex the

Page 77: MRCS preparation emrcs questions Lowerlimb

nerve knee Soleus Tibial

nerve Plantar flexor

Deep posterior compartment Muscle Nerve Action Flexor digitorum longus Tibial Flexes the lateral four toes Flexor hallucis longus Tibial Flexes the great toe Tibialis posterior Tibial Plantar flexor, inverts the foot A 23 year old lady is undergoing a trendelenberg procedure for varicose veins. During the dissection of the saphenofemoral junction, which of the following is most liable to injury?

A. Superficial circumflex iliac artery

B. Superficial circumflex iliac vein

C. Femoral artery

D. Femoral nerve

E. Deep external pudendal artery

Theme from September 2011 exam The deep external pudendal artery runs under the long saphenous vein close to its origin and may be injured. It is at greatest risk of injury during the flush ligation of the saphenofemoral junction. Provided an injury is identified and vessel ligated, injury is seldom associated with any serious adverse sequelae.

Saphenous vein Long saphenous vein This vein may be harvested for triple or quadruple bypass surgery

• Originates at the 1st digit where the dorsal vein merges with the dorsal venous arch of the foot

• Passes anterior to the medial malleolus and runs up the medial side of the leg • At the knee, it runs over the posterior border of the medial epicondyle of the

femur bone • Then passes laterally to lie on the anterior surface of the thigh before entering

an opening in the fascia lata called the saphenous opening • It joins with the femoral vein in the region of the femoral triangle at the

saphenofemoral junction

Page 78: MRCS preparation emrcs questions Lowerlimb

Tributaries

• Medial marginal • Superficial epigastric • Superficial iliac circumflex • Superficial external pudendal veins

Short saphenous vein

• Originates at the 5th digit where the dorsal vein merges with the dorsal venous arch of the foot, which attaches to the great saphenous vein.

• It passes around the lateral aspect of the foot (inferior and posterior to the lateral malleolus) and runs along the posterior aspect of the leg (with the sural nerve)

• Passes between the heads of the gastrocnemius muscle, and drains into the popliteal vein, approximately at or above the level of the knee joint.

A 52 year female post hysterectomy attends clinic. She reports pain and reduced sensation over the medial aspect of her thigh. Clinically thigh adduction is weak. What is the most likely nerve injury?

A. Obturator nerve

B. Sciatic nerve

C. Femoral nerve

D. L3 cord compression

E. Deep peroneal nerve

The obturator nerve supplies sensation to the medial aspect of the thigh and causes adduction and internal rotation of the thigh. Injury occurs during pelvic or abdominal surgery. L3 cord compression is unlikely.

Obturator nerve The obturator nerve arises from L2, L3 and L4 by branches from the ventral divisions of each

Page 79: MRCS preparation emrcs questions Lowerlimb

of these nerve roots. L3 forms the main contribution and the second lumbar branch is occasionally absent. These branches unite in the substance of psoas major, descending vertically in its posterior part to emerge from its medial border at the lateral margin of the sacrum. It then crosses the sacroiliac joint to enter the lesser pelvis, it descends on obturator internus to enter the obturator groove. In the lesser pelvis the nerve lies lateral to the internal iliac vessels and ureter, and is joined by the obturator vessels lateral to the ovary or ductus deferens. Supplies

• Medial compartment of thigh • Muscles supplied: external obturator, adductor longus, adductor brevis, adductor

magnus (not the lower part-sciatic nerve), gracilis • The cutaneous branch is often absent. When present, it passes between gracilis and

adductor longus near the middle part of the thigh, and supplies the skin and fascia of the distal two thirds of the medial aspect.

Obturator canal

• Connects the pelvis and thigh: contains the obturator artery, vein, nerve which divides into anterior and posterior branches.

Cadaveric cross section demonstrating relationships of the obturator nerve

Image sourced from Wikipedia

With regard to the sciatic nerve which statement is false?

A. It is derived from L4 to S3

Page 80: MRCS preparation emrcs questions Lowerlimb

B. It contains nerve roots from the posterior division of the lumbosacral plexus only

C. It divides to give the tibial nerve and common peroneal nerve

D. It provides cutaneous sensation to the posterior aspect of the thigh

E. It provides cutaneous sensation to the entire lower leg with the exception of its medial aspect

It is derived from both anterior and posterior divisions of the lumbosacral plexus. The sciatic nerve is the longest and widest nerve in the human body. It is particularly susceptible to trauma in the posterior approach to the hip.

Sciatic nerve Origin Spinal nerves L4 - S3 Articular Branches Hip joint Muscular branches in upper leg

• Semitendinosus • Semimembranosus • Biceps femoris • Part of adductor magnus

Cutaneous sensation • Posterior aspect of thigh • Gluteal region • Entire lower leg (except the medial aspect)

Terminates At the upper part of the popliteal fossa by dividing into the tibial and peroneal nerves

• The nerve to the short head of the biceps femoris comes from the common peroneal part of the sciatic and the other muscular branches arise from the tibial portion.

• The tibial nerve goes on to innervate all muscles of the foot except the extensor digitorum brevis (which is innervated by the common peroneal nerve).

Which of the following is a content of the adductor canal?

A. Saphenous nerve

B. Sural nerve

Page 81: MRCS preparation emrcs questions Lowerlimb

C. Femoral nerve

D. Profunda branch of the femoral artery

E. Saphenous vein

It contains the saphenous nerve and the superficial branch of the femoral artery.

Adductor canal

• Also called Hunter's or subsartorial canal

• Immediately distal to the apex of the femoral triangle, lying in the middle third of the thigh. Canal terminates at the adductor hiatus.

Borders Contents

Laterally Vastus medialis muscle Saphenous nerve

Posteriorly Adductor longus, adductor magnus Superficial femoral artery

Roof Sartorius Superficial femoral vein

In the image below the sartorius muscle is removed to expose the canal contents

Page 82: MRCS preparation emrcs questions Lowerlimb

Image sourced from Wikipedia

A 56 year old lady with metastatic breast cancer develops an oestolytic deposit in the proximal femur. One morning whilst getting out of bed she notices severe groin pain. X-rays show that the lesser trochanter has been avulsed. Which muscle is the most likely culprit?

A. Vastus lateralis

B. Psoas major

C. Piriformis

D. Gluteus maximus

E. Gluteus medius

The psoas major inserts into the lesser trochanter and contracts when raising the trunk

Page 83: MRCS preparation emrcs questions Lowerlimb

from the supine position. When oestolytic lesions are present in the femur the lesser trochanter may be avulsed.

Psoas Muscle Origin The deep part originates from the transverse processes of the five lumbar vertebrae, the superficial part originates from T12 and the first 4 lumbar vertebrae. Insertion Lesser trochanter of the femur. Innervation Anterior rami of L1 to L3. Action Flexion and external rotation of the hip. Bilateral contraction can raise the trunk from the supine position. A 34 year old man is injured by farm machinery and sustains a laceration at the superolateral aspect of the popliteal fossa. The medial aspect of biceps femoris is lacerated. Which of the following underlying structures is at greatest risk of injury?

A. Gracilis

B. Sural nerve

C. Nerve to semimembranosus

D. Popliteal artery

E. Common peroneal nerve

The common peroneal nerve lies under the medial aspect of biceps femoris and is therefore at greatest risk of injury. The tibial nerve may also be damaged in such an injury (but is not listed here). The sural nerve branches off more inferiorly.

Common peroneal nerve Derived from the dorsal divisions of the sacral plexus (L4, L5, S1 and S2). This nerve supplies the skin and fascia of the anterolateral surface of the leg and the dorsum of the foot. It also innervates the muscles of the anterior and peroneal compartments of the leg, extensor digitorum brevis as well as the knee, ankle and foot joints. It is laterally placed within the sciatic nerve. From the bifurcation of the sciatic nerve it passes inferolaterally in the lateral and proximal part of the popliteal fossa, under the cover of biceps femoris and its tendon. To reach the posterior aspect of the fibular head. It ends by dividing into the deep and superficial peroneal nerves at the point

Page 84: MRCS preparation emrcs questions Lowerlimb

where it winds around the lateral surface of the neck of the fibula in the body of peroneus longus, approximately 2cm distal to the apex of the head of the fibula. It is palpable posterior to the head of the fibula. Branches In the thigh Nerve to the short head of biceps

Articular branch (knee) In the popliteal fossa Lateral cutaneous nerve of the calf Neck of fibula Superficial and deep peroneal nerves A laceration to the upper lateral margin of the popliteal fossa may injure which of the following nerves?

A. Common peroneal nerve

B. Sural nerve

C. Sciatic nerve

D. Saphenous nerve

E. Tibial nerve

The sural nerve exits at the lower latero-medial aspect of the fossa and is more at risk in short saphenous vein surgery. The tibial nerve lies more medially and is even less likely to be injured in this location.

Popliteal fossa Boundaries of the popliteal fossa

Laterally Biceps femoris above, lateral head of gastrocnemius and plantaris below

Medially Semimembranosus and semitendinosus above, medial head of gastrocnemius below

Floor Popliteal surface of the femur, posterior ligament of knee joint and popliteus muscle

Roof Superficial and deep fascia

Page 85: MRCS preparation emrcs questions Lowerlimb

Image showing the popliteal fossa

Image sourced from Wikipedia

Contents

• Popliteal artery and vein • Small saphenous vein • Common peroneal nerve • Tibial nerve • Posterior cutaneous nerve of the thigh • Genicular branch of the obturator nerve • Lymph nodes

n elderly lady falls and lands on her hip. On examination her hip is tender to palpation and x-rays are taken. There are concerns that she may have an intertrochanteric fracture. What is the normal angle between the femoral neck and the femoral shaft?

A. 90o

B. 105o

C. 80o

D. 130o

Page 86: MRCS preparation emrcs questions Lowerlimb

E. 180o

The normal angle between the femoral head and shaft is 130o. Changes to this angle may occur as a result of disease or pathology and should be investigated.

Hip joint

• Head of femur articulates with acetabulum of the pelvis • Both covered by articular hyaline cartilage • The acetabulum forms at the union of the ilium, pubis, and ischium • The triradiate cartilage (Y-shaped growth plate) separates the pelvic bones • The acetabulum holds the femoral head by the acetabular labrum • Normal angle between femoral head and femoral shaft is 130o

Ligaments

• Transverse ligament: joints anterior and posterior ends of the articular cartilage • Head of femur ligament (ligamentum teres): acetabular notch to the fovea. Contains

arterial supply to head of femur in children.

Image sourced from Wikipedia

Page 87: MRCS preparation emrcs questions Lowerlimb

Image sourced from Wikipedia

Extracapsular ligaments

• Iliofemoral ligament: inverted Y shape. Anterior iliac spine to the trochanteric line • Pubofemoral ligament: acetabulum to lesser trochanter • Ischiofemoral ligament: posterior support. Ischium to greater trochanter.

Blood supply Medial circumflex femoral and lateral circumflex femoral arteries (Branches of profunda femoris) 2 anastomoses: Cruciate and the trochanteric anastomoses (provides most of the blood to the head of the femur) Hence the need for hemiarthroplasty when there is a displaced femoral head fracture. These anastomoses exist between the femoral artery or profunda femoris and the gluteal vessels. An 18 year old athlete attends orthopaedic clinic reporting pain and swelling over the medial aspect of the knee joint. The pain occurs when climbing the stairs, but is not present when walking on flat ground. Clinically there is pain over the medial, proximal tibia and the McMurray test is negative. What is the most likely cause of this patient's symptoms?

A. Anterior cruciate ligament tear

Page 88: MRCS preparation emrcs questions Lowerlimb

B. Prepatellar bursitis

C. Medial meniscus injury

D. Pes Anserinus Bursitis

E. Fracture of tibia

Pes anserinus: GOOSE'S FOOT Combination of sartorius, gracilis and semitendinous tendons inserting into the anteromedial proximal tibia. Pes Anserinus Bursitis is common in sportsmen due to overuse injuries. The main sign is of pain in the medial proximal tibia. As the McMurray test is negative, medial meniscal injury is excluded.

Sartorius

• Longest strap muscle in the body • Most superficial muscle in the anterior compartment of the thigh

Origin Anterior superior iliac spine Insertion Medial surface of the of the body of the tibia (upper part). It inserts

anterior to gracilis and semitendinosus Nerve Supply

Femoral nerve (L2,3)

Action • Flexor of the hip and knee, slight abducts the thigh and rotates it laterally

• It assists with medial rotation of the tibia on the femur. For example it would play a pivotal role in placing the right heel onto the left knee ( and vice versa)

Important relations

The middle third of this muscle, and its strong underlying fascia forms the roof of the adductor canal , in which lie the femoral vessels, the saphenous nerve and the nerve to vastus medialis.

Which of the following nerves innervates the long head of the biceps femoris muscle?

A. Inferior gluteal nerve

B. Tibial nerve

C. Superior gluteal nerve

Page 89: MRCS preparation emrcs questions Lowerlimb

D. Common peroneal nerve

E. Obturator nerve

The short head of biceps femoris, which may occasionally be absent, is innervated by the common peroneal component of the sciatic nerve. The long head is innervated by the tibial nerve.

Biceps femoris The biceps femoris is one of the hamstring group of muscles located in the posterior upper thigh. It has two heads. Long head

Origin Ischial tuberosity

Insertion Fibular head

Action Knee flexion, lateral rotation tibia, extension hip

Innervation Tibial nerve (L5, S1, S2)

Arterial supply

Profunda femoris artery, inferior gluteal artery, and the superior muscular branches of popliteal artery

Image demonstrating the biceps femoris muscle, with the long head outlined

Page 90: MRCS preparation emrcs questions Lowerlimb

Image sourced from Wikipedia

Short head

Page 91: MRCS preparation emrcs questions Lowerlimb

Origin Lateral lip of linea aspera, lateral supracondylar ridge of femur

Insertion Fibular head

Action Knee flexion, lateral rotation tibia

Innervation Common peroneal nerve (L5, S1, S2)

Arterial supply

Profunda femoris artery, inferior gluteal artery, and the superior muscular branches of popliteal artery

Which of the following bones is related to the cuboid at its distal articular surface?

A. All metatarsals

B. 5th metatarsal

C. Calcaneum

D. Medial cuneiform

E. 3rd metatarsal

Theme from April 2012 Exam The cuboid is located at the lateral aspect of the foot between the calcaneus posteriorly and the 4th and 5th metatarsals distally.

Foot- anatomy Arches of the foot The foot is conventionally considered to have two arches.

• The longitudinal arch is higher on the medial than on the lateral side. The posterior part of the calcaneum forms a posterior pillar to support the arch. The lateral part of this structure passes via the cuboid bone and the lateral two metatarsal bones. The medial part of this structure is more important. The head of the talus marks the summit of this arch, located between the sustentaculum tali and the navicular bone. The anterior pillar of the medial arch is composed of the navicular bone, the three cuneiforms and the medial three metatarsal bones.

Page 92: MRCS preparation emrcs questions Lowerlimb

• The transverse arch is situated on the anterior part of the tarsus and the posterior part of the metatarsus. The cuneiforms and metatarsal bases narrow inferiorly, which contributes to the shape of the arch.

Intertarsal joints

Sub talar joint Formed by the cylindrical facet on the lower surface of the body of the talus and the posterior facet on the upper surface of the calcaneus. The facet on the talus is concave anteroposteriorly, the other is convex. The synovial cavity of this joint does not communicate with any other joint.

Talocalcaneonavicular joint

The anterior part of the socket is formed by the concave articular surface of the navicular bone, posteriorly by the upper surface of the sustentaculum tali. The talus sits within this socket

Calcaneocuboid joint Highest point in the lateral part of the longitudinal arch. The lower aspect of this joint is reinforced by the long plantar and plantar calcaneocuboid ligaments.

Transverse tarsal joint The talocalcaneonavicular joint and the calcaneocuboid joint extend accross the tarsus in an irregular transverse plane, between the talus and calcaneus behind and the navicular and cuboid bones in front. This plane is termed the transverse tarsal joint.

Cuneonavicular joint Formed between the convex anterior surface of the navicular bone and the concave surface of the the posterior ends of the three cuneiforms.

Intercuneiform joints Between the three cuneiform bones.

Cuneocuboid joint Between the circular facets on the lateral cuneiform bone and the cuboid. This joint contributes to the tarsal part of the transverse arch.

A detailed knowledge of the joints is not required for MRCS Part A. However, the contribution they play to the overall structure of the foot should be appreciated Ligaments of the ankle joint and foot

Page 93: MRCS preparation emrcs questions Lowerlimb

Image sourced from Wikipedia

Muscles of the foot

Muscle Origin Insertion Nerve supply

Action

Abductor hallucis

Medial side of the calcaneus, flexor retinaculum, plantar aponeurosis

Medial side of the base of the proximal phalanx

Medial plantar nerve

Abducts the great toe

Flexor digitorum brevis

Medial process of the calcaneus, plantar eponeurosis.

Via 4 tendons into the middle phalanges of the lateral 4 toes.

Medial plantar nerve

Flexes all the joints of the lateral 4 toes except for the interphalangeal joint.

Abductor digit minimi

From the tubercle of the calcaneus and from the plantar aponeurosis

Together with flexor digit minimi brevis into the lateral side of the base of the proximal

Lateral plantar nerve

Abducts the little toe at the metatarsophalangeal joint

Page 94: MRCS preparation emrcs questions Lowerlimb

phalanx of the little toe

Flexor hallucis brevis

From the medial side of the plantar surface of the cuboid bone, from the adjacent part of the lateral cuneiform bone and from the tendon of tibialis posterior.

Into the proximal phalanx of the great toe, the tendon contains a sesamoid bone

Medial plantar nerve

Flexes the metatarsophalangeal joint of the great toe.

Adductor hallucis

Arises from two heads. The oblique head arises from the sheath of the peroneus longus tendon, and from the plantar surfaces of the bases of the 2nd, 3rd and 4th metatarsal bones. The transverse head arises from the plantar surface of the lateral 4 metatarsophalangeal joints and from the deep transverse metatarsal ligament.

Lateral side of the base of the proximal phalanx of the great toe.

Lateral plantar nerve

Adducts the great toe towards the second toe. Helps maintain the transverse arch of the foot.

Extensor digitorum brevis

On the dorsal surface of the foot from the upper surface of the calcaneus and its associated fascia

Via four thin tendons which run forward and medially to be inserted into the medial four toes. The lateral three tendons join with hoods of extensor digitorum longus.

Deep peroneal

Extend the metatarsophalangeal joint of the medial four toes. It is unable to extend the interphalangeal joint without the assistance of the lumbrical muscles.

Detailed knowledge of the foot muscles are not needed for the MRCS part A Nerves in the foot

Page 95: MRCS preparation emrcs questions Lowerlimb

Lateral plantar nerve Passes anterolaterally towards the base of the 5th metatarsal between flexor digitorum brevis and flexor accessorius. On the medial aspect of the lateral plantar artery. At the base of the 5th metatarsal it splits into superficial and deep branches. Medial plantar nerve Passes forwards with the medial plantar artery under the cover of the flexor retinaculum to the interval between abductor hallucis and flexor digitorum brevis on the sole of the foot. Plantar arteries Arise under the cover of the flexor retinaculum, midway between the tip of the medial malleolus and the most prominent part of the medial side of the heel.

• Medial plantar artery. Passes forwards medial to medial plantar nerve in the space between abductor hallucis and flexor digitorum brevis.Ends by uniting with a branch of the 1st plantar metatarsal artery.

• Lateral plantar artery. Runs obliquely across the sole of the foot. It lies lateral to the lateral plantar nerve. At the base of the 5th metatarsal bone it arches medially across the foot on the metatarsals

Dorsalis pedis artery This vessel is a direct continuation of the anterior tibial artery. It commences on the front of the ankle joint and runs to the proximal end of the first metatarsal space. Here is gives off the arcuate artery and continues forwards as the first dorsal metatarsal artery. It is accompanied by two veins throughout its length. It is crossed by the extensor hallucis brevis A 40-year-old man presents with pain in his lower back and 'sciatica' for the past three days. He describes bending down to pick up a washing machine when he felt 'something go'. He now has severe pain radiating from his back down the right leg. On examination he describes paraesthesia over the anterior aspect of the right knee and the medial aspect of his calf. Power is intact and the right knee reflex is diminished. The femoral stretch test is positive on the right side. Which nerve or nerve root is most likely to be affected?

A. Common peroneal nerve

B. Lateral cutaneous nerve of the thigh

C. L5

D. L3

E. L4

Page 96: MRCS preparation emrcs questions Lowerlimb

Prolapsed disc A prolapsed lumbar disc usually produces clear dermatomal leg pain associated with neurological deficits. Features

• Leg pain usually worse than back • Pain often worse when sitting

The table below demonstrates the expected features according to the level of compression:

L3 nerve root compression

Sensory loss over anterior thigh Weak quadriceps Reduced knee reflex Positive femoral stretch test

L4 nerve root compression

Sensory loss anterior aspect of knee Weak quadriceps Reduced knee reflex Positive femoral stretch test

L5 nerve root compression

Sensory loss dorsum of foot Weakness in foot and big toe dorsiflexion Reflexes intact Positive sciatic nerve stretch test

S1 nerve root compression

Sensory loss posterolateral aspect of leg and lateral aspect of foot Weakness in plantar flexion of foot Reduced ankle reflex Positive sciatic nerve stretch test

Management

• Similar to that of other musculoskeletal lower back pain: analgesia, physiotherapy, exercises

• Persistent symptoms, muscular weakness, bladder or bowel dysfunction are indications for urgent MRI scanning to delineate the disease extent to allow surgical planning

• Plain spinal x-rays have no useful role in establishing the extent of disk disease

Onto which of the following structures does the anterior cruciate ligament insert?

A. Posterolateral aspect of the lateral femoral condyle

Page 97: MRCS preparation emrcs questions Lowerlimb

B. Posteromedial aspect of the lateral femoral condyle

C. Posterolateral aspect of the medial femoral condyle

D. Posteromedial aspect of the medial femoral condyle

E. None of the above

The anterior cruciate ligament is attached to the anterior intercondylar area of the tibia. Is then passes posterolaterally to insert into the posteromedial aspect of the lateral femoral condyle.

Knee joint The knee joint is a synovial joint, the largest and most complicated. It consists of two condylar joints between the femure and tibia and a sellar joint between the patella and the femur. The tibiofemoral articular surfaces are incongruent, however, this is improved by the presence of the menisci. The degree of congruence is related to the anatomical position of the knee joint and is greatest in full extension. Knee joint compartments

Tibiofemoral • Comprise of the patella/femur joint, lateral and medial compartments (between femur condyles and tibia)

• Synovial membrane and cruciate ligaments partially separate the medial and lateral compartments

Patellofemoral • Ligamentum patellae • Actions: provides joint stability in full extension

Fibrous capsule The capsule of the knee joint is a complex, composite structure with contributions from adjacent tendons.

Anterior fibres

The capsule does not pass proximal to the patella. It blends with the tendinous expansions of vastus medialis and lateralis

Posterior fibres

These fibres are vertical and run from the posterior surface of the femoral condyles to the posterior aspect of the tibial condyle

Medial Attach to the femoral and tibial condyles beyond their articular margins,

Page 98: MRCS preparation emrcs questions Lowerlimb

fibres blending with the tibial collateral ligament

Lateral fibres

Attach to the femur superior to popliteus, pass over its tendon to head of fibula and tibial condyle

Bursae

Anterior • Subcutaneous prepatellar bursa; between patella and skin • Deep infrapatellar bursa; between tibia and patellar ligament • Subcutaneous infrapatellar bursa; between distal tibial tuberosity and skin

Laterally • Bursa between lateral head of gastrocnemius and joint capsule • Bursa between fibular collateral ligament and tendon of biceps femoris • Bursa between fibular collateral ligament and tendon of popliteus

Medially • Bursa between medial head of gastrocnemius and the fibrous capsule • Bursa between tibial collateral ligament and tendons of sartorius, gracilis

and semitendinosus • Bursa between the tendon of semimembranosus and medial tibial condyle

and medial head of gastrocnemius

Posterior Highly variable and inconsistent

Ligaments

Medial collateral ligament

Medial epicondyle femur to medial tibial condyle: valgus stability

Lateral collateral ligament

Lateral epicondyle femur to fibula head: varus stability

Anterior cruciate ligament

Anterior tibia to lateral intercondylar notch femur: prevents tibia sliding anteriorly

Posterior cruciate ligament

Posterior tibia to medial intercondylar notch femur: prevents tibia sliding posteriorly

Patellar ligament Central band of the tendon of quadriceps femoris, extends from patella to tibial tuberosity

Page 99: MRCS preparation emrcs questions Lowerlimb

Image sourced from Wikipedia

Page 100: MRCS preparation emrcs questions Lowerlimb

Image sourced from Wikipedia

Menisci Medial and lateral menisci compensate for the incongruence of the femoral and tibial condyles. Composed of fibrous tissue. Medial meniscus is attached to the tibial collateral ligament. Lateral meniscus is attached to the loose fibres at the lateral edge of the joint and is separate from the fibular collateral ligament. The lateral meniscus is crossed by the popliteus tendon. Nerve supply The knee joint is supplied by the femoral, tibial and common peroneal divisions of the sciatic and by a branch from the obturator nerve. Hip pathology pain may be referred to the knee. Blood supply Genicular branches of the femoral artery, popliteal and anterior tibial arteries all supply the knee joint.

A 40 year old lady presents with varicose veins, these are found to originate from the short saphenous vein. As the vein is mobilised close to its origin which of the following structures is at greatest risk of injury?

A. Sciatic nerve

B. Sural nerve

C. Common peroneal nerve

D. Tibial nerve

E. Popliteal artery

The sural nerve is closely related and damage to this structure is a major cause of litigation. The other structures may all be injured but the risks are lower.

Popliteal fossa Boundaries of the popliteal fossa

Page 101: MRCS preparation emrcs questions Lowerlimb

Laterally Biceps femoris above, lateral head of gastrocnemius and plantaris below

Medially Semimembranosus and semitendinosus above, medial head of gastrocnemius below

Floor Popliteal surface of the femur, posterior ligament of knee joint and popliteus muscle

Roof Superficial and deep fascia

Image showing the popliteal fossa

Image sourced from Wikipedia

Contents

• Popliteal artery and vein • Small saphenous vein • Common peroneal nerve • Tibial nerve • Posterior cutaneous nerve of the thigh • Genicular branch of the obturator nerve • Lymph nodes

A 72 year old man with non reconstructible arterial disease is undergoing an above knee amputation. The posterior compartment muscles are divided. Which of the following muscles does not lie in the posterior compartment of the thigh?

A. Biceps femoris

B. Quadriceps femoris

Page 102: MRCS preparation emrcs questions Lowerlimb

C. Semitendinosus

D. Semimembranosus

E. None of the above

The quadriceps femoris lies in the anterior compartment.

Fascial compartments of the leg Compartments of the thigh Formed by 3 septae passing from the femur to the fascia lata. Compartment Nerve Muscles Blood supply Anterior compartment

Femoral • Iliacus • Tensor fasciae latae • Sartorius • Quadriceps femoris

Femoral artery

Medial compartment

Obturator • Adductor longus/magnus/brevis

• Gracilis • Obturator externus

Profunda femoris artery and obturator artery

Posterior compartment

Sciatic • Semimembranosus • Semitendinosus • Biceps femoris

Branches of Profunda femoris artery

Compartments of the lower leg Separated by the interosseous membrane (anterior and posterior compartments), anterior fascial septum (separate anterior and lateral compartments) and posterior fascial septum (separate lateral and posterior compartments)

Compartment Nerve Muscles Blood supply

Anterior compartment

Deep peroneal nerve

• Tibialis anterior • Extensor digitorum longus • Extensor hallucis longus • Peroneus tertius

Anterior tibial artery

Posterior compartment

Tibial • Muscles: deep and superficial compartments (separated by deep transverse fascia)

• Deep: Flexor hallucis longus, Flexor digitalis longus, Tibialis

Posterior tibial

Page 103: MRCS preparation emrcs questions Lowerlimb

posterior, Popliteus • Superficial: Gastrocnemius,

Soleus, Plantaris

Lateral compartment

Superficial peroneal

• Peroneus longus/brevis Anterior tibial

Which of the following structures is not closely related to the posterior tibial artery?

A. Soleus posteriorly

B. Tibial nerve medially

C. Deep peroneal nerve laterally

D. Flexor hallucis longus postero-inferiorly

E. Popliteus

The deep peroneal nerve lies in the anterior compartment. The tibial nerve lies medially. At its termination it lies deep to the flexor retinaculum.

Posterior tibial artery

• Larger terminal branch of the popliteal artery • Terminates by dividing into the medial and lateral plantar arteries • Accompanied by two veins throughout its length • Position of the artery corresponds to a line drawn from the lower angle of the

popliteal fossa, at the level of the neck of the fibula, to a point midway between the medial malleolus and the most prominent part of the heel

Relations of the posterior tibial artery Proximal to distal Anteriorly Tibialis posterior

Flexor digitorum longus Posterior surface of tibia and ankle joint

Posterior Tibial nerve 2.5 cm distal to its origin Fascia overlying the deep muscular layer Proximal part covered by gastrocnemius and soleus Distal part covered by skin and fascia

A 30 year old man presents with back pain and the surgeon tests the ankle reflex. Which of the following nerve roots are tested in this manoeuvre?

A. S3 and S4

Page 104: MRCS preparation emrcs questions Lowerlimb

B. L4 and L5

C. L3 and L4

D. S1 and S2

E. S4 only

Theme from April 2012 Exam

Ankle reflex The ankle reflex is elicited by tapping the Achilles tendon with a tendon hammer. It tests the S1 and S2 nerve roots. It is typically delayed in L5 and S1 disk prolapses. Which of the following structures is not closely related to the piriformis muscle?

A. Superior gluteal nerve

B. Sciatic nerve

C. Inferior gluteal artery

D. Inferior gluteal nerve

E. Medial femoral circumflex artery

Nerve supply of lateral hip rotators Piriformis: ventral rami S1, S2 Obturator internus: nerve to obturator internus Superior gemellus: nerve to obturator internus Inferior gemellus: nerve to quadratus femoris Quadrator femoris: nerve to quadrator femoris The piriformis muscle is an important anatomical landmark in the gluteal region. The following structures are closely related:

• Sciatic nerve • Inferior gluteal artery and nerve • Superior gluteal artery and nerve

The medial femoral circumflex artery runs deep to quadratus femoris.

Page 105: MRCS preparation emrcs questions Lowerlimb

Gluteal region Gluteal muscles

• Gluteus maximus: inserts to gluteal tuberosity of the femur and iliotibial tract • Gluteus medius: attach to lateral greater trochanter • Gluteus minimis: attach to anterior greater trochanter • All extend and abduct the hip

Deep lateral hip rotators

• Piriformis • Gemelli • Obturator internus • Quadratus femoris

Nerves Superior gluteal nerve (L5, S1) • Gluteus medius

• Gluteus minimis • Tensor fascia lata

Inferior gluteal nerve Gluteus maximus Damage to the superior gluteal nerve will result in the patient developing a Trendelenberg gait. Affected patients are unable to abduct the thigh at the hip joint. During the stance phase, the weakened abductor muscles allow the pelvis to tilt down on the opposite side. To compensate, the trunk lurches to the weakened side to attempt to maintain a level pelvis throughout the gait cycle. The pelvis sags on the opposite side of the lesioned superior gluteal nerve. A 77 year old man with symptoms of intermittent claudication is due to have his ankle brachial pressure indices measured. The vessel is impalpable. Which of the following tendinous structures lies medial to it, that may facilitate its identification?

A. Extensor digitorum longus tendon

B. Peroneus tertius tendon

C. Extensor hallucis longus tendon

D. Extensor digitorum brevis tendon

E. Flexor digitorum longus tendon

Page 106: MRCS preparation emrcs questions Lowerlimb

The extensor hallucis longus tendon lies medial to the dorsalis pedis artery.

Foot- anatomy Arches of the foot The foot is conventionally considered to have two arches.

• The longitudinal arch is higher on the medial than on the lateral side. The posterior part of the calcaneum forms a posterior pillar to support the arch. The lateral part of this structure passes via the cuboid bone and the lateral two metatarsal bones. The medial part of this structure is more important. The head of the talus marks the summit of this arch, located between the sustentaculum tali and the navicular bone. The anterior pillar of the medial arch is composed of the navicular bone, the three cuneiforms and the medial three metatarsal bones.

• The transverse arch is situated on the anterior part of the tarsus and the posterior part of the metatarsus. The cuneiforms and metatarsal bases narrow inferiorly, which contributes to the shape of the arch.

Intertarsal joints

Sub talar joint Formed by the cylindrical facet on the lower surface of the body of the talus and the posterior facet on the upper surface of the calcaneus. The facet on the talus is concave anteroposteriorly, the other is convex. The synovial cavity of this joint does not communicate with any other joint.

Talocalcaneonavicular joint

The anterior part of the socket is formed by the concave articular surface of the navicular bone, posteriorly by the upper surface of the sustentaculum tali. The talus sits within this socket

Calcaneocuboid joint Highest point in the lateral part of the longitudinal arch. The lower aspect of this joint is reinforced by the long plantar and plantar calcaneocuboid ligaments.

Transverse tarsal joint The talocalcaneonavicular joint and the calcaneocuboid joint extend accross the tarsus in an irregular transverse plane, between the talus and calcaneus behind and the navicular and cuboid bones in front. This plane is termed the transverse tarsal joint.

Cuneonavicular joint Formed between the convex anterior surface of the navicular bone and the concave surface of the the posterior ends of the three cuneiforms.

Intercuneiform joints Between the three cuneiform bones.

Page 107: MRCS preparation emrcs questions Lowerlimb

Cuneocuboid joint Between the circular facets on the lateral cuneiform bone and the cuboid. This joint contributes to the tarsal part of the transverse arch.

A detailed knowledge of the joints is not required for MRCS Part A. However, the contribution they play to the overall structure of the foot should be appreciated Ligaments of the ankle joint and foot

Image sourced from Wikipedia

Muscles of the foot

Muscle Origin Insertion Nerve supply

Action

Abductor hallucis

Medial side of the calcaneus, flexor retinaculum, plantar aponeurosis

Medial side of the base of the proximal phalanx

Medial plantar nerve

Abducts the great toe

Flexor digitorum brevis

Medial process of the calcaneus, plantar eponeurosis.

Via 4 tendons into the middle phalanges of the lateral 4

Medial plantar nerve

Flexes all the joints of the lateral 4 toes except for the interphalangeal joint.

Page 108: MRCS preparation emrcs questions Lowerlimb

toes.

Abductor digit minimi

From the tubercle of the calcaneus and from the plantar aponeurosis

Together with flexor digit minimi brevis into the lateral side of the base of the proximal phalanx of the little toe

Lateral plantar nerve

Abducts the little toe at the metatarsophalangeal joint

Flexor hallucis brevis

From the medial side of the plantar surface of the cuboid bone, from the adjacent part of the lateral cuneiform bone and from the tendon of tibialis posterior.

Into the proximal phalanx of the great toe, the tendon contains a sesamoid bone

Medial plantar nerve

Flexes the metatarsophalangeal joint of the great toe.

Adductor hallucis

Arises from two heads. The oblique head arises from the sheath of the peroneus longus tendon, and from the plantar surfaces of the bases of the 2nd, 3rd and 4th metatarsal bones. The transverse head arises from the plantar surface of the lateral 4 metatarsophalangeal joints and from the deep transverse metatarsal ligament.

Lateral side of the base of the proximal phalanx of the great toe.

Lateral plantar nerve

Adducts the great toe towards the second toe. Helps maintain the transverse arch of the foot.

Extensor digitorum brevis

On the dorsal surface of the foot from the upper surface of the calcaneus and its associated fascia

Via four thin tendons which run forward and medially to be inserted into the medial four toes. The lateral three tendons join with hoods of

Deep peroneal

Extend the metatarsophalangeal joint of the medial four toes. It is unable to extend the interphalangeal joint without the assistance of the lumbrical muscles.

Page 109: MRCS preparation emrcs questions Lowerlimb

extensor digitorum longus.

Detailed knowledge of the foot muscles are not needed for the MRCS part A Nerves in the foot Lateral plantar nerve Passes anterolaterally towards the base of the 5th metatarsal between flexor digitorum brevis and flexor accessorius. On the medial aspect of the lateral plantar artery. At the base of the 5th metatarsal it splits into superficial and deep branches. Medial plantar nerve Passes forwards with the medial plantar artery under the cover of the flexor retinaculum to the interval between abductor hallucis and flexor digitorum brevis on the sole of the foot. Plantar arteries Arise under the cover of the flexor retinaculum, midway between the tip of the medial malleolus and the most prominent part of the medial side of the heel.

• Medial plantar artery. Passes forwards medial to medial plantar nerve in the space between abductor hallucis and flexor digitorum brevis.Ends by uniting with a branch of the 1st plantar metatarsal artery.

• Lateral plantar artery. Runs obliquely across the sole of the foot. It lies lateral to the lateral plantar nerve. At the base of the 5th metatarsal bone it arches medially across the foot on the metatarsals

Dorsalis pedis artery This vessel is a direct continuation of the anterior tibial artery. It commences on the front of the ankle joint and runs to the proximal end of the first metatarsal space. Here is gives off the arcuate artery and continues forwards as the first dorsal metatarsal artery. It is accompanied by two veins throughout its length. It is crossed by the extensor hallucis brevis The integrity of which muscle is assessed by the Trendelenberg test?

A. Sartorius

B. Quadratus femoris

C. Semimembranosus

D. Gluteus medius

E. Piriformis

Page 110: MRCS preparation emrcs questions Lowerlimb

Theme from 2011 Exam

Trendelenberg test Injury or division of the superior gluteal nerve results in a motor deficit that consists of weakened abduction of the thigh by gluteus medius, a disabling gluteus medius limp and a compensatory list of the body weakened gluteal side. The compensation results in a gravitational shift so that the body is supported on the unaffected limb. When a person is asked to stand on one leg the gluteus medius usually contracts as soon as the contralateral leg leaves the floor, preventing the pelvis from dipping towards the unsupported side. When a person with paralysis of the superior gluteal nerve is asked to stand on one leg, the pelvis on the unsupported side descends, indicating that the gluteus medius on the affected side is weak or non functional ( a positive Trendelenberg test). A 78 year old man presents with symptoms consistent with intermittent claudication. To assess the severity of his disease you decide to measure his ankle brachial pressure index. To do this you will identify the dorsalis pedis artery. Which of the following statements relating to this vessel is false?

A. It originates from the peroneal artery

B. It is crossed by the tendon of extensor hallucis brevis

C. Two veins are usually closely related to it

D. It passes under the inferior extensor retinaculum

E. The tendon of extensor hallucis longus lies medial to it.

The dorsalis pedis artery is a direct continuation of the anterior tibial artery.

Foot- anatomy Arches of the foot The foot is conventionally considered to have two arches.

• The longitudinal arch is higher on the medial than on the lateral side. The posterior part of the calcaneum forms a posterior pillar to support the arch. The lateral part of this structure passes via the cuboid bone and the lateral two metatarsal bones. The medial part of this structure is more important. The head of the talus marks the summit of this arch, located between the sustentaculum tali and the navicular bone. The anterior pillar of the medial arch is composed of the navicular bone, the three cuneiforms and the medial three metatarsal bones.

Page 111: MRCS preparation emrcs questions Lowerlimb

• The transverse arch is situated on the anterior part of the tarsus and the posterior part of the metatarsus. The cuneiforms and metatarsal bases narrow inferiorly, which contributes to the shape of the arch.

Intertarsal joints

Sub talar joint Formed by the cylindrical facet on the lower surface of the body of the talus and the posterior facet on the upper surface of the calcaneus. The facet on the talus is concave anteroposteriorly, the other is convex. The synovial cavity of this joint does not communicate with any other joint.

Talocalcaneonavicular joint

The anterior part of the socket is formed by the concave articular surface of the navicular bone, posteriorly by the upper surface of the sustentaculum tali. The talus sits within this socket

Calcaneocuboid joint Highest point in the lateral part of the longitudinal arch. The lower aspect of this joint is reinforced by the long plantar and plantar calcaneocuboid ligaments.

Transverse tarsal joint The talocalcaneonavicular joint and the calcaneocuboid joint extend accross the tarsus in an irregular transverse plane, between the talus and calcaneus behind and the navicular and cuboid bones in front. This plane is termed the transverse tarsal joint.

Cuneonavicular joint Formed between the convex anterior surface of the navicular bone and the concave surface of the the posterior ends of the three cuneiforms.

Intercuneiform joints Between the three cuneiform bones.

Cuneocuboid joint Between the circular facets on the lateral cuneiform bone and the cuboid. This joint contributes to the tarsal part of the transverse arch.

A detailed knowledge of the joints is not required for MRCS Part A. However, the contribution they play to the overall structure of the foot should be appreciated Ligaments of the ankle joint and foot

Page 112: MRCS preparation emrcs questions Lowerlimb

Image sourced from Wikipedia

Muscles of the foot

Muscle Origin Insertion Nerve supply

Action

Abductor hallucis

Medial side of the calcaneus, flexor retinaculum, plantar aponeurosis

Medial side of the base of the proximal phalanx

Medial plantar nerve

Abducts the great toe

Flexor digitorum brevis

Medial process of the calcaneus, plantar eponeurosis.

Via 4 tendons into the middle phalanges of the lateral 4 toes.

Medial plantar nerve

Flexes all the joints of the lateral 4 toes except for the interphalangeal joint.

Abductor digit minimi

From the tubercle of the calcaneus and from the plantar aponeurosis

Together with flexor digit minimi brevis into the lateral side of the base of the proximal

Lateral plantar nerve

Abducts the little toe at the metatarsophalangeal joint

Page 113: MRCS preparation emrcs questions Lowerlimb

phalanx of the little toe

Flexor hallucis brevis

From the medial side of the plantar surface of the cuboid bone, from the adjacent part of the lateral cuneiform bone and from the tendon of tibialis posterior.

Into the proximal phalanx of the great toe, the tendon contains a sesamoid bone

Medial plantar nerve

Flexes the metatarsophalangeal joint of the great toe.

Adductor hallucis

Arises from two heads. The oblique head arises from the sheath of the peroneus longus tendon, and from the plantar surfaces of the bases of the 2nd, 3rd and 4th metatarsal bones. The transverse head arises from the plantar surface of the lateral 4 metatarsophalangeal joints and from the deep transverse metatarsal ligament.

Lateral side of the base of the proximal phalanx of the great toe.

Lateral plantar nerve

Adducts the great toe towards the second toe. Helps maintain the transverse arch of the foot.

Extensor digitorum brevis

On the dorsal surface of the foot from the upper surface of the calcaneus and its associated fascia

Via four thin tendons which run forward and medially to be inserted into the medial four toes. The lateral three tendons join with hoods of extensor digitorum longus.

Deep peroneal

Extend the metatarsophalangeal joint of the medial four toes. It is unable to extend the interphalangeal joint without the assistance of the lumbrical muscles.

Detailed knowledge of the foot muscles are not needed for the MRCS part A Nerves in the foot

Page 114: MRCS preparation emrcs questions Lowerlimb

Lateral plantar nerve Passes anterolaterally towards the base of the 5th metatarsal between flexor digitorum brevis and flexor accessorius. On the medial aspect of the lateral plantar artery. At the base of the 5th metatarsal it splits into superficial and deep branches. Medial plantar nerve Passes forwards with the medial plantar artery under the cover of the flexor retinaculum to the interval between abductor hallucis and flexor digitorum brevis on the sole of the foot. Plantar arteries Arise under the cover of the flexor retinaculum, midway between the tip of the medial malleolus and the most prominent part of the medial side of the heel.

• Medial plantar artery. Passes forwards medial to medial plantar nerve in the space between abductor hallucis and flexor digitorum brevis.Ends by uniting with a branch of the 1st plantar metatarsal artery.

• Lateral plantar artery. Runs obliquely across the sole of the foot. It lies lateral to the lateral plantar nerve. At the base of the 5th metatarsal bone it arches medially across the foot on the metatarsals

Dorsalis pedis artery This vessel is a direct continuation of the anterior tibial artery. It commences on the front of the ankle joint and runs to the proximal end of the first metatarsal space. Here is gives off the arcuate artery and continues forwards as the first dorsal metatarsal artery. It is accompanied by two veins throughout its length. It is crossed by the extensor hallucis brevis Which of the following statements relating to the knee joint is false?

A. It is the largest synovial joint in the body

B. When the knee is fully extended all ligaments of the knee joint are taut

C. Rupture of the anterior cruciate ligament may result in haemarthrosis

D. The posterior aspect of the patella is extrasynovial

E. The joint is innervated by the femoral, sciatic and obturator nerves

The posterior aspect is intrasynovial and the knee itself comprises the largest synovial joint in the body. It may swell considerably following trauma such as ACL injury. Which may be extremely painful owing to rich innervation from femoral, sciatic and ( a smaller)

Page 115: MRCS preparation emrcs questions Lowerlimb

contribution from the obturator nerve. During full extension all ligaments are taut and the knee is locked.

Knee joint The knee joint is a synovial joint, the largest and most complicated. It consists of two condylar joints between the femure and tibia and a sellar joint between the patella and the femur. The tibiofemoral articular surfaces are incongruent, however, this is improved by the presence of the menisci. The degree of congruence is related to the anatomical position of the knee joint and is greatest in full extension. Knee joint compartments

Tibiofemoral • Comprise of the patella/femur joint, lateral and medial compartments (between femur condyles and tibia)

• Synovial membrane and cruciate ligaments partially separate the medial and lateral compartments

Patellofemoral • Ligamentum patellae • Actions: provides joint stability in full extension

Fibrous capsule The capsule of the knee joint is a complex, composite structure with contributions from adjacent tendons.

Anterior fibres

The capsule does not pass proximal to the patella. It blends with the tendinous expansions of vastus medialis and lateralis

Posterior fibres

These fibres are vertical and run from the posterior surface of the femoral condyles to the posterior aspect of the tibial condyle

Medial fibres

Attach to the femoral and tibial condyles beyond their articular margins, blending with the tibial collateral ligament

Lateral fibres

Attach to the femur superior to popliteus, pass over its tendon to head of fibula and tibial condyle

Bursae

Anterior • Subcutaneous prepatellar bursa; between patella and skin • Deep infrapatellar bursa; between tibia and patellar ligament • Subcutaneous infrapatellar bursa; between distal tibial tuberosity and skin

Laterally • Bursa between lateral head of gastrocnemius and joint capsule • Bursa between fibular collateral ligament and tendon of biceps femoris

Page 116: MRCS preparation emrcs questions Lowerlimb

• Bursa between fibular collateral ligament and tendon of popliteus

Medially • Bursa between medial head of gastrocnemius and the fibrous capsule • Bursa between tibial collateral ligament and tendons of sartorius, gracilis

and semitendinosus • Bursa between the tendon of semimembranosus and medial tibial condyle

and medial head of gastrocnemius

Posterior Highly variable and inconsistent

Ligaments

Medial collateral ligament

Medial epicondyle femur to medial tibial condyle: valgus stability

Lateral collateral ligament

Lateral epicondyle femur to fibula head: varus stability

Anterior cruciate ligament

Anterior tibia to lateral intercondylar notch femur: prevents tibia sliding anteriorly

Posterior cruciate ligament

Posterior tibia to medial intercondylar notch femur: prevents tibia sliding posteriorly

Patellar ligament Central band of the tendon of quadriceps femoris, extends from patella to tibial tuberosity

Page 117: MRCS preparation emrcs questions Lowerlimb

Image sourced from Wikipedia

Page 118: MRCS preparation emrcs questions Lowerlimb

Image sourced from Wikipedia

Menisci Medial and lateral menisci compensate for the incongruence of the femoral and tibial condyles. Composed of fibrous tissue. Medial meniscus is attached to the tibial collateral ligament. Lateral meniscus is attached to the loose fibres at the lateral edge of the joint and is separate from the fibular collateral ligament. The lateral meniscus is crossed by the popliteus tendon. Nerve supply The knee joint is supplied by the femoral, tibial and common peroneal divisions of the sciatic and by a branch from the obturator nerve. Hip pathology pain may be referred to the knee. Blood supply Genicular branches of the femoral artery, popliteal and anterior tibial arteries all supply the knee joint.

hich of the following does not exit the pelvis through the greater sciatic foramen?

A. Superior gluteal artery

B. Internal pudendal vessels

C. Sciatic nerve

D. Obturator nerve

E. Inferior gluteal nerve

The obturator nerve exits through the obturator foramen.

Greater sciatic foramen Contents

Nerves • Sciatic Nerve • Superior and Inferior Gluteal Nerves • Internal Pudendal Nerve • Posterior Femoral Cutaneous Nerve

Page 119: MRCS preparation emrcs questions Lowerlimb

• Nerve to Quadratus Femoris • Nerve to Obturator internus

Vessels • Superior Gluteal Artery and vein • Inferior Gluteal Artery and vein • Internal Pudendal Artery and vein

Piriformis The piriformis is a landmark for identifying structures passing out of the sciatic notch

• Above piriformis: Superior gluteal vessels • Below piriformis: Inferior gluteal vessels, sciatic nerve (10% pass through it, <1%

above it), posterior cutaneous nerve of the thigh

Greater sciatic foramen boundaries

Anterolaterally Greater sciatic notch of the ilium

Posteromedially Sacrotuberous ligament

Inferior Sacrospinous ligament and the ischial spine

Superior Anterior sacroiliac ligament

The greater sciatic foramen

Page 120: MRCS preparation emrcs questions Lowerlimb

Image sourced from Wikipedia

Structures passing between both foramina (Medial to lateral)

• Pudendal nerve • Internal pudendal artery • Nerve to obturator internus

Contents of the lesser sciatic foramen

• Tendon of the obturator internus • Pudendal nerve • Internal pudendal artery and vein • Nerve to the obturator internus

A 78 year old man is undergoing a femoro-popliteal bypass graft. The operation is not progressing well and the surgeon is complaining of poor access. Retraction of which of the following structures will improve access to the femoral artery in the groin?

Page 121: MRCS preparation emrcs questions Lowerlimb

A. Quadriceps

B. Adductor longus

C. Adductor magnus

D. Pectineus

E. Sartorius

At the lower border of the femoral triangle the femoral artery passes under the sartorius muscle. This can be retracted to improve access.

Femoral triangle anatomy Boundaries

Superiorly Inguinal ligament

Laterally Sartorius

Medially Adductor longus

Floor Iliopsoas, adductor longus and pectineus

Roof • Fascia lata and Superficial fascia • Superficial inguinal lymph nodes (palpable below the inguinal ligament) • Great saphenous vein

Page 122: MRCS preparation emrcs questions Lowerlimb

Image sourced from Wikipedia

Contents

• Femoral vein (medial to lateral) • Femoral artery-pulse palpated at the mid inguinal point • Femoral nerve • Deep and superficial inguinal lymph nodes • Lateral cutaneous nerve • Great saphenous vein • Femoral branch of the genitofemoral nerve

A builder falls off a ladder whilst laying roof tiles. He sustains a burst fracture of L3. The MRI scan shows complete cord transection at this level as a result of the injury. Which clinical sign will not be present?

A. Flaccid paralysis of the legs

B. Extensor plantar response

C. Sensory loss in the legs

D. Incontinence

Page 123: MRCS preparation emrcs questions Lowerlimb

E. Loss of patellar tendon reflex

In lower motor neuron lesions everything is reduced

The main purpose of this question is to differentiate the features of an UMN lesion and a LMN lesion. The features of a LMN lesion include:

• Flaccid paralysis of muscles supplied • Atrophy of muscles supplied. • Loss of reflexes of muscles supplied. • Muscles fasciculation

For lesions below L1 LMN signs will occur. Hence in an L3 lesion, there will be loss of the patella reflex but there will be no extensor plantar reflex.

Spinal cord

• Located in a canal within the vertebral column that affords it structural support.

• Rostrally is continues to the medulla oblongata of the brain and caudally it tapers at a level corresponding to the L1-2 interspace (in the adult), a central structure, the filum terminale anchors the cord to the first coccygeal vertebra.

• The spinal cord is characterised by cervico-lumbar enlargements and these, broadly speaking, are the sites which correspond to the brachial and lumbar plexuses respectively.

There are some key points to note when considering the surgical anatomy of the spinal cord: * During foetal growth the spinal cord becomes shorter than the spinal canal, hence the adult site of cord termination at the L1-2 level. * Due to growth of the vertebral column the spine segmental levels may not always correspond to bony landmarks as they do in the cervical spine. * The spinal cord is incompletely divided into two symmetrical halves by a dorsal median sulcus and ventral median fissure. Grey matter surrounds a central canal that is continuous rostrally with the ventricular system of the CNS. * The grey matter is sub divided cytoarchitecturally into Rexeds laminae. * Afferent fibres entering through the dorsal roots usually terminate near their point of entry but may travel for varying distances in Lissauers tract. In this way they may establish synaptic connections over several levels

Page 124: MRCS preparation emrcs questions Lowerlimb

* At the tip of the dorsal horn are afferents associated with nociceptive stimuli. The ventral horn contains neurones that innervate skeletal muscle. The key point to remember when revising CNS anatomy is to keep a clinical perspective in mind. So it is worth classifying the ways in which the spinal cord may become injured. These include:

• Trauma either direct or as a result of disc protrusion • Neoplasia either by direct invasion (rare) or as a result of pathological

vertebral fracture • Inflammatory diseases such as Rheumatoid disease, or OA (formation of

osteophytes compressing nerve roots etc. • Vascular either as a result of stroke (rare in cord) or as complication of aortic

dissection • Infection historically diseases such as TB, epidural abscesses.

The anatomy of the cord will, to an extent dictate the clinical presentation. Some points/ conditions to remember:

• Brown- Sequard syndrome-Hemisection of the cord producing ipsilateral loss of proprioception and upper motor neurone signs, plus contralateral loss of pain and temperature sensation. The explanation of this is that the fibres decussate at different levels.

• Lesions below L1 will tend to present with lower motor neurone signs

66 year old man with peripheral vascular disease is undergoing a below knee amputation. In which of the lower leg compartments does peroneus brevis lie?

A. Lateral compartment

B. Anterior compartment

C. Superficial posterior compartment

D. Deep posterior compartment

E. None of the above

The interosseous membrane separates the anterior and posterior compartments. The deep and superficial compartments are separated by the deep transverse fascia. The peroneus brevis is part of the lateral compartment.

Fascial compartments of the leg Compartments of the thigh Formed by 3 septae passing from the femur to the fascia lata. Compartment Nerve Muscles Blood supply

Page 125: MRCS preparation emrcs questions Lowerlimb

Anterior compartment

Femoral • Iliacus • Tensor fasciae latae • Sartorius • Quadriceps femoris

Femoral artery

Medial compartment

Obturator • Adductor longus/magnus/brevis

• Gracilis • Obturator externus

Profunda femoris artery and obturator artery

Posterior compartment

Sciatic • Semimembranosus • Semitendinosus • Biceps femoris

Branches of Profunda femoris artery

Compartments of the lower leg Separated by the interosseous membrane (anterior and posterior compartments), anterior fascial septum (separate anterior and lateral compartments) and posterior fascial septum (separate lateral and posterior compartments)

Compartment Nerve Muscles Blood supply

Anterior compartment

Deep peroneal nerve

• Tibialis anterior • Extensor digitorum longus • Extensor hallucis longus • Peroneus tertius

Anterior tibial artery

Posterior compartment

Tibial • Muscles: deep and superficial compartments (separated by deep transverse fascia)

• Deep: Flexor hallucis longus, Flexor digitalis longus, Tibialis posterior, Popliteus

• Superficial: Gastrocnemius, Soleus, Plantaris

Posterior tibial

Lateral compartment

Superficial peroneal

• Peroneus longus/brevis Anterior tibial

A 70 year old man is due to undergo an arterial bypass procedure for claudication and foot ulceration. The anterior tibial artery will form the site of the distal arterial anastomosis. Which of the following structures is not closely related to it?

A. Interosseous membrane

B. Deep peroneal nerve

Page 126: MRCS preparation emrcs questions Lowerlimb

C. Tibialis posterior

D. Extensor hallucis longus

E. Dorsalis pedis artery

As an artery of the anterior compartment, the anterior tibial artery is closely related to tibialis anterior.

Anterior tibial artery

• Begins opposite the distal border of popliteus • Terminates in front of the ankle, continuing as the dorsalis pedis artery • As it descends it lies on the interosseous membrane, distal part of the tibia and

front of the ankle joint • Passes between the tendons of flexor digitorum and flexor hallucis longus

distally • It is related to the deep peroneal nerve, it lies anterior to the middle third of the

vessel and lateral to it in the lower third

Which of the following muscles does not cause lateral rotation of the hip?

A. Obturator internus

B. Quadratus femoris

C. Gemellus inferior

D. Piriformis

E. Pectineus

Mnemonic lateral hip rotators: P-GO-GO-Q (top to bottom) Piriformis Gemellus superior Obturator internus Gemellus inferior Obturator externus Quadratus femoris

Page 127: MRCS preparation emrcs questions Lowerlimb

Pectineus adducts and medially rotates the femur.

Hip joint

• Head of femur articulates with acetabulum of the pelvis • Both covered by articular hyaline cartilage • The acetabulum forms at the union of the ilium, pubis, and ischium • The triradiate cartilage (Y-shaped growth plate) separates the pelvic bones • The acetabulum holds the femoral head by the acetabular labrum • Normal angle between femoral head and femoral shaft is 130o

Ligaments

• Transverse ligament: joints anterior and posterior ends of the articular cartilage • Head of femur ligament (ligamentum teres): acetabular notch to the fovea. Contains

arterial supply to head of femur in children.

Image sourced from Wikipedia

Page 128: MRCS preparation emrcs questions Lowerlimb

Image sourced from Wikipedia

Extracapsular ligaments

• Iliofemoral ligament: inverted Y shape. Anterior iliac spine to the trochanteric line • Pubofemoral ligament: acetabulum to lesser trochanter • Ischiofemoral ligament: posterior support. Ischium to greater trochanter.

Blood supply Medial circumflex femoral and lateral circumflex femoral arteries (Branches of profunda femoris) 2 anastomoses: Cruciate and the trochanteric anastomoses (provides most of the blood to the head of the femur) Hence the need for hemiarthroplasty when there is a displaced femoral head fracture. These anastomoses exist between the femoral artery or profunda femoris and the gluteal vessels. Which of the following structures does not pass anterior to the lateral malleolus?

A. Anterior tibial artery

B. Extensor digitorum longus

C. Tibialis anterior

Page 129: MRCS preparation emrcs questions Lowerlimb

D. Peroneus brevis

E. Peroneus tertius

Peroneus brevis passes posterior to the lateral malleolus.

Lateral malleolus Structures posterior to the lateral malleolus and superficial to superior peroneal retinaculum

• Sural nerve • Short saphenous vein

Structures posterior to the lateral malleolus and deep to superior peroneal retinaculum

• Peroneus longus tendon • Peroneus brevis tendon

The calcaneofibular ligament is attached at the lateral malleolus Which of the following statements relating to sartorius is untrue?

A. It is supplied by the femoral nerve

B. It forms the lateral boundary of the femoral triangle

C. The middle third forms the roof of the adductor canal

D. It is a flexor of the hip and knee

E. It inserts into the medial femoral condyle

It inserts into the medial aspect of the upper part of the tibia.

Sartorius

• Longest strap muscle in the body • Most superficial muscle in the anterior compartment of the thigh

Origin Anterior superior iliac spine Insertion Medial surface of the of the body of the tibia (upper part). It inserts

anterior to gracilis and semitendinosus

Page 130: MRCS preparation emrcs questions Lowerlimb

Nerve Supply

Femoral nerve (L2,3)

Action • Flexor of the hip and knee, slight abducts the thigh and rotates it laterally

• It assists with medial rotation of the tibia on the femur. For example it would play a pivotal role in placing the right heel onto the left knee ( and vice versa)

Important relations

The middle third of this muscle, and its strong underlying fascia forms the roof of the adductor canal , in which lie the femoral vessels, the saphenous nerve and the nerve to vastus medialis.

Which of the following structures are not closely related to the adductor longus muscle?

A. Long saphenous vein

B. Tendon of iliacus

C. The profunda branch of the femoral artery

D. Pectineus muscle

E. Femoral nerve

Femoral triangle: Adductor longus medially Inguinal ligament superiorly Sartorius muscle laterally

Adductor longus forms the medial border of the femoral triangle. It is closely related to the long saphenous vein which overlies it and the profunda branch of the femoral artery. The femoral nerve is related to it inferiorly. However, the tendon of iliacus inserts proximally and is not in contact with adductor longus.

Adductor longus

Origin Anterior body of pubis

Page 131: MRCS preparation emrcs questions Lowerlimb

Insertion Middle third of linea aspera

Action Adducts and flexes the thigh, medially rotate the hip

Innervation Anterior division of obturator nerve (L2, L3, L4)

The schematic image below demonstrates the relationship of the adductor muscles

Image sourced from Wikipedia

Which of the following muscles is not within the posterior compartment of the lower leg?

A. Peroneus brevis

B. Flexor digitalis longus

C. Soleus

D. Popliteus

E. Flexor hallucis longus

Page 132: MRCS preparation emrcs questions Lowerlimb

Theme from 2007 Exam Peroneus brevis lies in the lateral compartment.

Fascial compartments of the leg Compartments of the thigh Formed by 3 septae passing from the femur to the fascia lata. Compartment Nerve Muscles Blood supply Anterior compartment

Femoral • Iliacus • Tensor fasciae latae • Sartorius • Quadriceps femoris

Femoral artery

Medial compartment

Obturator • Adductor longus/magnus/brevis

• Gracilis • Obturator externus

Profunda femoris artery and obturator artery

Posterior compartment

Sciatic • Semimembranosus • Semitendinosus • Biceps femoris

Branches of Profunda femoris artery

Compartments of the lower leg Separated by the interosseous membrane (anterior and posterior compartments), anterior fascial septum (separate anterior and lateral compartments) and posterior fascial septum (separate lateral and posterior compartments)

Compartment Nerve Muscles Blood supply

Anterior compartment

Deep peroneal nerve

• Tibialis anterior • Extensor digitorum longus • Extensor hallucis longus • Peroneus tertius

Anterior tibial artery

Posterior compartment

Tibial • Muscles: deep and superficial compartments (separated by deep transverse fascia)

• Deep: Flexor hallucis longus, Flexor digitalis longus, Tibialis posterior, Popliteus

• Superficial: Gastrocnemius, Soleus, Plantaris

Posterior tibial

Page 133: MRCS preparation emrcs questions Lowerlimb

Lateral compartment

Superficial peroneal

• Peroneus longus/brevis Anterior tibial

The femoral nerve is transected by a rather careless surgeon during a botched femoro-popliteal bypass operation. Which of the following actions will be impaired?

A. Extension of the great toe

B. Adduction of the thigh

C. Flexion of the knee joint

D. Extension of the knee joint

E. Eversion of the foot

The femoral nerve supplies the quadriceps muscle which is responsible for extension at the knee joint.

Femoral nerve

Root values L2, 3, 4

Innervates • Pectineus • Sartorius • Quadriceps femoris • Vastus lateralis/medialis/intermedius

Branches • Medial cutaneous nerve of thigh • Saphenous nerve • Intermediate cutaneous nerve of thigh

Path Penetrates psoas major and exits the pelvis by passing under the inguinal ligament to enter the femoral triangle, lateral to the femoral artery and vein.

Page 134: MRCS preparation emrcs questions Lowerlimb

Image sourced from Wikipedia

Mnemonic for femoral nerve supply (don't) M I S V Q Scan for PE M edial cutaneous nerve of the thigh I ntermediate cutaneous nerve of the thigh S aphenous nerve V astus Q uadriceps femoris S artorius PE ectineus Which of the following structures are at risk of direct injury following a fracture dislocation of the femoral condyles?

A. Popliteal artery

Page 135: MRCS preparation emrcs questions Lowerlimb

B. Sciatic nerve

C. Plantaris muscle

D. Tibial artery

E. Tibial nerve

The heads of gastrocnemius will contract to pull the fracture segment posteriorly. The popliteal artery lies against the bone and may be damaged or compressed.

Popliteal fossa Boundaries of the popliteal fossa

Laterally Biceps femoris above, lateral head of gastrocnemius and plantaris below

Medially Semimembranosus and semitendinosus above, medial head of gastrocnemius below

Floor Popliteal surface of the femur, posterior ligament of knee joint and popliteus muscle

Roof Superficial and deep fascia

Image showing the popliteal fossa

Image sourced from Wikipedia

Page 136: MRCS preparation emrcs questions Lowerlimb

Contents

• Popliteal artery and vein • Small saphenous vein • Common peroneal nerve • Tibial nerve • Posterior cutaneous nerve of the thigh • Genicular branch of the obturator nerve • Lymph nodes